You are on page 1of 91

9R SOLUTIONS FROM AAMC 

Physical Sciences

1. 1. A solution contains 0.1 M Mg2+(aq), 0.1 M Ca2+(aq), and 0.1 M Sr2+(aq). All
three ions can be precipitated if which two reagents are added to the solution?
A
) Reagents 1 and 2
B
) Reagents 1 and 3

Mg2+ can be precipitated from a solution of Reagent 1 only. Ca2+ can be


precipitated from a solution of reagents 3 or 4, and Sr2+ can be precipitated from a
solution of reagents 2, 3, or 4. All three ions can be precipitated from a solution
containing Reagent 1 and reagents 3 or 4. The ions will precipitate from a solution
containing reagents 1 and 3. Thus, B is the best answer.

C
) Reagents 2 and 3

D
) Reagents 2 and 4

Solution

Guess
2. A test tube contains 2 mL of 0.1 M Ca2+(aq). A precipitate will most likely
form if which of the following reagents is added to the tube?

A
) 1.0 M HCl(aq)

B
) 1.0 M NaOH(aq)

C
) 1.0 M H2SO4(aq)

D
) 1.0 M Na2CO3(aq)

Ca2+ forms a precipitate when either Reagent 3, a solution of (NH4)2C2O4, or


Reagent 4, a solution of (NH4)2CO3, is added to the Ca(NO3)2 solution. A precipitate
would form when a solution of Na2CO3 is added. Na2CO3, like (NH4)2CO3, is a
source of CO32– and its addition would result in the formation of the insoluble CaCO3.
Thus, D is the best answer.

Solution

Guess
3. The students added Reagent 2 to a test tube containing 0.1 M Ca2+(aq) and
0.1 M Sr2+(aq). Which of the following procedures will best enable the students to
recover a fairly pure sample of SrSO4(s) from this mixture?
A
) Allowing the water to evaporate and collecting the solid that remains in
the tube

B
) Pouring the mixture through a filter, collecting the insoluble substance,
and allowing the water to evaporate from the insoluble substance

When Reagent 2 is added to the Ca2+ solution, the resulting solution is clear. When
Reagent 2 is added to the Sr2+ solution, a white precipitate is generated. It follows
that after the addition of Reagent 2 to a solution containing Ca2+ and Sr2+, the
Ca2+ would remain in the solution and the Sr2+ would precipitate out as SrSO4. Pure
SrSO4 could be obtained by isolating the solid by filtration and allowing the water to
evaporate from the solid. Thus, B is the best answer.

C
) Pouring the mixture through a filter, collecting the filtrate containing
the soluble substance, then allowing the water to evaporate from the filtrate

D
) Adding Reagent 3, pouring the mixture through a filter, collecting the
filtrate containing the soluble substance, and allowing the water to evaporate from
the filtrate

Solution

Guess
4. The information in Table 1 suggests that which of the following substances has
the smallest Ksp?

A
) MgSO4

B
) MgC2O4

C
) CaSO4

D
) CaC2O4

The smaller the Ksp of a substance, the lower the solubility of the substance. MgSO4,
MgC2O4, and CaSO4 all remain dissolved, but CaC2O4 does not. Therefore, CaC2O4
has the lowest solubility and the smallest Ksp. Thus, D is the best answer.

Solution

Guess
5. A solution contains either Ca2+(aq) or Sr2+(aq). Which of the following
actions will best enable the students to identify the ion in the solution?

A
) Performing a flame test
B
) Adding Reagent 1

C
) Adding Reagent 2

When Reagent 2 is added to the Ca2+ solution, the resulting solution is clear. When
Reagent 2 is added to the Sr2+ solution, a white precipitate forms. The students will
be able to determine which ion is present by adding Reagent 2. Thus, C is the best
answer.

D
) Adding Reagent 3

Solution

Guess
6. Which of the following graphs best illustrates how charge accumulates on the
plates of the capacitor after the switch is closed?

A
)

B
)

As time increases, charging must increase as well. Further, the charging of a


capacitor is an exponentially decreasing process that must reach a constant value
after a long time. The graph in B most clearly shows this behavior. Thus, B is the best
answer.
C
)

D
)
Solution

Guess
7. If the speed of the charged particle described in the passage is increased by a
factor of 2, the electrical force on the particle will:

A
) decrease by a factor of 2.

B
) remain the same.

Electrical force depends on the particle’s charge and the strength of the electric field
experienced by the particle, not on the particle’s speed. Thus, B is the best answer.

C
) increase by a factor of 2.

D
) increase by a factor of 4.
Solution
Guess
8. Making which of the following changes to a circuit element will increase the
capacitance of the capacitor described in the passage?

A
) Replacing the 500-Ω resistor with a 250-Ω resistor

B
) Replacing the 10-V battery with a 20-V battery

C
) Increasing the separation of the capacitor plates

D
) Increasing the area of the capacitor plates

Capacitance C depends on geometric factors only, and in the case of parallel plates,
C is proportional to the plate area and inversely proportional to the separation
distance of the plates. Thus, D is the best answer.

Solution

Guess
9. A charged particle with a mass of m and a charge of q is injected midway
between the plates of a capacitor that has a uniform electric field of E. What is the
acceleration of this particle due to the electric field?

A
) Eq/m

The force on the charge is qE and force is also ma. Setting qE = ma and solving for
acceleration a yields

Thus, A is the best answer.

B
) Em/q

C
) mq/E

D
) Emq

Solution

Guess
10. Another capacitor, identical to the original, is added in series to the circuit
described in the passage. Compared to the original circuit, the equivalent
capacitance of the new circuit is:

A
) 1/2 as great.

When in series, capacitors C1 and C2 add by the inverse rule

Therefore, if C1 = C2 = C, then

Thus, A is the best answer.

B
) the same.

C
) 2 times as great.

D
) 4 times as great.

Solution

Guess
11. Which of the following best describes the motion of a negatively charged
particle after it has been injected between the plates of a charged, parallel-plate
capacitor? (Note: Assume that the area between the plates is a vacuum.)

A
) It moves with constant speed toward the positive plate.

B
) It moves with constant speed toward the negative plate.

C
) It accelerates toward the positive plate.

Opposites attract, so the negatively charged particle will move toward the positive
plate. Because there is a constant force qE on the particle, it will accelerate toward
the positive plate. Thus, C is the best answer.

D
) It accelerates toward the negative plate.

Solution

Guess
12. What type of intermolecular interaction can ETOH undergo with water that
MTBE can NOT?

A
) van der Waals

B
) Dipole–dipole

C
) Hydrogen bonding
The –OH group in ETOH gives ETOH the capability of donating hydrogen bonds to
water. MTBE is incapable of donating hydrogen bonds to water. Thus, C is the best
answer.

D
) Covalent bonding

Solution

Guess
13. The formation of one mole of which oxygenate shown in Table 1 releases the
most energy?

A
) ETOH

B
) MTBE

C
) ETBE

D
) TAME

The formation of 1 mol of TAME gives off the most energy, 680 kJ. Thus, D is the best
answer.

Solution

Guess
14.

What are the coefficients for oxygen and carbon dioxide, respectively, if the equation
shown below is balanced?

1CH3OCH3(ℓ ) + ? O2(g) ® __ H2O(g) + ? CO2(g)

A
) 2 and 1

B
) 2 and 2

C
) 3 and 1

D
) 3 and 2

The six hydrogen atoms in CH3OCH3 on the left side of the equation must be
balanced by giving H2O on the right side of the equation a coefficient of 3. The two
carbon atoms in CH3OCH3 on the left side of the equation must be balanced by
giving CO2 on the right side of the equation a coefficient of 2. Now, the seven oxygen
atoms on the right side of the equation must be balanced by giving O2 on the left
side of the equation a coefficient of 3. Thus, D is the best answer.

Solution

Guess
15. Which of the following nonoxygenated analogs of MTBE is most likely to mimic
MTBE in its antiknock properties?

A
) C4H9Si(CH3)3

B
) C4H9N(CH3)2

C
) C4H9SCH3

Because sulfur is in the same group as oxygen in the periodic table, these elements
share many characteristics and it would be expected that replacing the O in MTBE
with S would result in a compound that would most likely mimic the antiknock
properties of MTBE. Thus, C is the best answer.
D
) C4H9P(CH3)2

Solution

Guess
16. The entropy change for the combustion reaction of gasoline is always greater
than zero because the:

A
) number of gaseous molecules in the products always exceeds the
number of gaseous molecules in the reactants.

Entropy is a measure of disorder. Gases have higher entropies than liquids, and
liquids have higher entropies than solids. The entropy of a reaction increases if the
number of moles of gaseous products exceeds the number of moles of gaseous
reactants. Thus, A is the best answer.

B
) enthalpy change is always positive.

C
) temperature of the combustion is always more than 100°C.

D
) free energy change is always positive.

Solution

Guess
17. Which compound shown in Table 1 evaporates fastest at 30°C?
A
) MTBE

B
) ETOH

The greater the vapor pressure, the greater the rate of evaporation. ETOH has the
highest vapor pressure at 25ºC. Thus, B is the best answer.

C
) ETBE

D
) TAME

Solution

Guess
18. If one mole of each additive shown in Table 1 undergoes complete
combustion, which compound requires the least amount of oxygen?

A
) MTBE

B
) ETOH

The compound with the lowest carbon-to-oxygen ratio will require the least amount
of oxygen for combustion. ETOH has only two carbon atoms and one oxygen atom.
Thus, B is the best answer.

C
) ETBE

D
) TAME

Solution

Guess
19. A collection of an unspecified number of neighboring gas columns, or pipes,
can reasonably be used to represent the layer of a star in which pressure waves
occur because the:

A
) harmonic frequencies of a pipe are independent of its diameter.
The passage gives the relationship between frequency and pipe geometry and shows
it being dependent on length only. This makes A correct and B incorrect. C is
eliminated because the velocity is explicitly given as a function of gas density. D is
incorrect because gas density falls with height. Thus, A is the best answer.

B
) harmonic frequencies of a pipe are independent of its length.

C
) speed of sound in gas confined to a pipe is independent of gas density.

D
) speed of sound propagating upward against gravity decreases with
height.

Solution

Guess
20. As an aid in identifying the various resonant pressure-wave frequencies in the
Sun and stars, one can use the fact that:

A
) the Doppler-shifted light is easily recognized, being polarized in a way
that is characteristic of hydrogen.

B
) the Doppler-shifted light stands out, being steadier in intensity than
the unshifted light emissions that accompany it.

C
) resonant frequencies are always separated by increments that are
equal to a basic number multiplied by an integer.

Only C is given as a possibility in the passage, namely, the quantization of the


resonant frequencies. This is in fact the basis for the analogy to the pipe. Thus, C is
the best answer.

D
) resonant frequencies in hydrogen gas depend strongly on its degree of
gas ionization, which, in turn, depends on temperature.

Solution

Guess
21. In the newer observational technique discussed, one makes use of the fact
that:

A
) the hydrogen gas in the observed stellar atmospheres is completely
ionized.
B
) stellar atmospheres are open to space, so that pressure and
temperature are independent of volume.

C
) pressure waves in stars propagate upward very slowly, generally at
about 1 m/s.

D
) light is absorbed or emitted whenever electrons move from one energy
level to another.

This item is fairly subtle in that only D is correct based on the passage. The gas is in
an excited state but not completely ionized, making A incorrect. B is entirely
incorrect, and C is true only for the gas velocity, not for the pressure wave velocity.
Thus, D is the best answer.

Solution

Guess
22. The surface temperature of one of the observed stars is 6000 K. The fraction
of its surface hydrogen atoms having electrons in energy level 2 then increases by
how much for each 1 K temperature rise?

A
) (6001/5999)6 1.002, so 2 parts in 1000

B
) (6001/6000)6 1.001, so 1 part in 1000

The passage states that the population increases with T6. For an initial surface
temperature of 6000 K, an increase in 1 K must result in a population increase that is
the ratio of the new temperature to the old temperature raised to the 6th power

Thus, B is the best answer.

C
) (274/273)6 1.02, so 2 parts in 100

D
) (461/460)

Solution

Guess
23. H2O is liquid at room temperature, whereas H2S, H2Se, and H2Te are all
gases. Which of the following best explains why H2O is liquid at room temperature?

A
) Hydrogen bonds form between H2O molecules.

H2O is capable of forming intermolecular hydrogen bonds, but H2S, H2Se, and H2Te
are not. Thus, A is the best answer.
B
) Oxygen lacks d orbitals.

C
) H2O has a lower molecular weight.

D
) H2O is more volatile.

Solution

Guess
24.

How do the pressures Pw and Pm compare, measured at the bottom of two identical
containers filled to the levels shown in the figure with water and mercury? (Note:
Density of water = 1 g/cm3; density of mercury = 14 g/cm3.)

A
) Pm = 2Pw

B
) Pm = 7Pw

Pressure is given by (density) • g • (height). Here, mercury’s density is given as 14


times that of water but the column is only half as high, making the pressure of the
mercury (14)(0.5) = 7 times more than that of the water. Thus, B is the best answer.

C
) Pm = 14Pw

D
) Pm = 28Pw

Solution

Guess
25. If the second floor and the top floor of a building are separated by a distance
of 100 m, what is the approximate difference between the air pressures of the two
levels? (Note: Air density = 1.2 kg/m3 and gravitational acceleration = 10 m/s2.
Neglect the compressibility of air.)

A
) 600 N/m2

B
) 800 N/m2

C
) 1000 N/m2

D
) 1200 N/m2

Pressure is given by (density) • g • (height) if density does not change. This is


approximately true for small atmospheric height differences, such as 100 m. The
pressure difference will be given by

(density) • g • (change in height)

or (1.2)(10)(100) = 1200 N/m2. Thus, D is the best answer.

Solution

Guess
26. What is the pH of a .001 M NaOH solution?

A
) .001

B
) 3

C
) 7

D
) 11

A .001 M NaOH solution has a pOH (–log [OH-]) of 3. The pH of the solution is
therefore 14 – 3 = 11. Thus, D is the best answer.

Solution

Guess
27. Why did the researchers choose helium as a buffer gas?

A
) It is chemically inert and has no effect on the rate.

Helium is chemically inert and does not affect the metal–hydrocarbon reactions.
Thus, A is the best answer.

B
) It catalyzes the reaction, increasing the rate.

C
) It collides with metal atoms, decreasing their kinetic energy and the
rate.

D
) Its atoms are approximately the same size as the metal atoms, and
they have no effect on the rate.
Solution

Guess
28. If the reaction tube described in the passage is 2 cm in diameter and an initial
helium–metal mixture displays ideal gas behavior, which of the following expressions
gives the number of moles of He in the tube at 1.1 torr and 298 K? (Note: R = 0.082 L
atm K–1 mol–1.)

A
) [(1.1)(π)(1)2(0.98)]/[(1000)(760)(0.082)(298)]

B
) [(1.1)(π)(1)(73)2(0.98)]/[(1000)(760)(0.082)(298)]

C
) [(1.1)(π)(2)2(73)(0.98)(4.0)]/[(1000)(760)(0.082)(298)]

D
) [(1.1)(π)(1)2(73)(0.98)]/[(1000)(760)(0.082)(298)]

According to the ideal gas law, PV = nRT or n = PV/RT. According to Dalton’s law of
partial pressures, PHe = Ptotal × mole fraction of He. The number of moles of He, n,
can be calculated using the following values in the ideal gas law:

P = 0.98 × 1.1 torr × 1 atm/760 torr

V = volume of the tube = π × (1 cm)2 × 73 cm × 1 L/1000 cm3

T = 298 K

Thus, D is the best answer.

Solution

Guess
29. What does a nonlinear plot of ln [M] versus t indicate about the concentration
of the hydrocarbon (HC) and the reaction mechanism?

A
)
The HC is not in excess, and the mechanism is
M + heat ® M* (fast)
M* + HC ® product (slow)

Because the plot of ln [M] versus t is nonlinear, it can be determined that the HC is
involved in the rate-determining (slow) step of the reaction. According to the
passage, when HC is in excess, a linear plot would be expected. Thus, A is the best
answer.

B
)
The HC is in excess, and the mechanism is
M + heat ® M* (fast)
M* + HC ® product (slow)

C
)
The HC is not in excess, and the mechanism is
M + heat ® M* (slow)
M* + HC ® product (fast)

D
)
The HC is in excess, and the mechanism is
M + heat ® M* (slow)
M* + HC ® product (fast)

Solution

Guess
30. According to Table 2 and information in the passage, the reactivity of platinum
(5d96s1) relative to gold (5d106s1) is attributable to the metal–HC interaction, which
involves:

A
) only valence s electrons.

B
) sd hybrid orbitals.

According to the passage, valence s and d orbitals of similar energies form sd hybrid
orbitals, enhancing the reaction. The valence s and d orbitals in platinum form sd
hybrid orbitals, increasing platinum’s reactivity toward the HC. Thus, B is the best
answer.

C
) sp hybrid orbitals.

D
) only valence d electrons.

Solution

Guess
31. The metals shown in Table 2 belong to which block of elements in the periodic
table?

A
) s

B
) p

C
) d
The metals in Table 2 are transition metals belonging to the d-block of the periodic
table. Thus, C is the best answer.

D
) f

Solution

Guess
32. Ta reacts slower than Nb (Table 2) because:

A
) the valence s orbitals of Ta have a much higher energy than do its
valence d orbitals.

B
) the valence s orbitals of Ta have a much lower energy than do its
valence d orbitals.

To form the hybrid orbitals, the valence s and d orbitals must have similar energies.
Ta reacts slowly, suggesting that the difference in its valence s and d orbitals’
energies is too great. The valence s orbital will be lower in energy. Thus, B is the best
answer.

C
) the valence s orbitals of Nb have a much higher energy than do its
valence d orbitals.

D
) Ta forms especially stable sd hybrid orbitals.

Solution

Guess
33. Which of the following expressions gives the percent mass of hafnium (Hf) in
an initial mixture of He–Hf at 0.5 torr?

A
) [(0.93)(4.0)(100)]/[(0.07)(178.5) + (0.93)(4.0)]

B
) [(90.07)(72)(100)]/[(0.07)(972) + (0.93)(2)]

C
) [(0.07)(178.5)(100)]/[(0.07)(178.5)+(0.93)(4.0)]

At 0.5 torr, the mole fraction of He is 0.93 and the mole fraction of Hf is 0.07.

% mass Hf = [mass Hf/(mass Hf + mass He)] × 100%

Substitute the following:


mass Hf = 0.07 mol Hf × 178.5 g/mol Hf

mass He = 0.93 mol He × 4.0 g/mol He

Thus, C is the best answer.

D
) [(0.07)(200.6)(100)]/[(0.07)(200.6) + (0.93)(4.0)]

Solution

Guess
34. In Experiment 1, the acceleration of the hub of the tire during the first 4 s is:

A
) a nonzero constant in the direction of the frictional force.

B
) a nonzero constant in the direction of the pulling force.

C
) increasing steadily as the pulling force increases.

D
) constant and zero.

In Experiment 1, the tire and axle are expected to remain at rest until the rope’s pull
exceeds the maximum static friction force between the tire and the road; then the
rope’s pull will decrease to a value equal to the kinetic friction force between the tire
and the road. The data in Table 1 shows an increasing force during the first 5 s, so for
the time interval 0–4 s, the rope’s pull must be balanced by static friction, leaving the
tire and axle at rest. Thus, D is the best answer.

Solution

Guess
35. What is the coefficient of static friction in Experiment 1?

A
) 1.5

The maximum static force is found in Table 1 to be 7500 N and the normal force is
(500 kg)•g, or 5000 N. Therefore, the coefficient of static friction is (7500/5000), or
1.5. Thus, A is the best answer.

B
) 1500

C
) 6000 N
D
) 7500 N

Solution

Guess
36. What is the coefficient of kinetic friction in Experiment 1?

A
) 1500 N

B
) 6000 N

C
) 1.2

The kinetic force on the sliding tire is found in Table 1 to be 6000 N (see time interval
6–9 s) and the normal force is (500 kg)•g, or 5000 N. Therefore, the coefficient of
kinetic friction is (6000/5000), or 1.2. Thus, C is the best answer.

D
) 1.5

Solution

Guess
37. The initial translational kinetic energy of the wheel system in Experiment 2
(just before applying the brakes):

A
) is less than the magnitude of work required to stop the tire.

B
) is equal to the magnitude of work required to stop the tire.

The work–energy theorem states that the change in the kinetic energy of a system is
equal to the work done on the system. The work done on the system is that needed
to stop the tire, and the change in the kinetic energy is equal to the initial kinetic
energy (because the final kinetic energy is zero). Thus, B is the best answer.

C
) is greater than the magnitude of work required to stop the tire.

D
) cannot be determined from the information given.

Solution

Guess
38. If a tire with a radius of 0.5 m is rolling with an angular frequency of 30 rad/s,
how far will the axle travel in 2 s?

A
) 5m
B
) 10 m

C
) 20 m

D
) 30 m

The speed of the axle v is related to the tire’s angular frequency by

v = rω

Because the distance d traveled is

d = vt

in 2 s, the axle will travel a distance of

d = v•t = rω•t = (0.5 m)(30 rad/s)(2 s) = 30 m

Thus, D is the best answer.

Solution

Guess
39. Which of the following solutions has the lowest pH at 25°C?

A
) 0.1 M HNO2(aq)

The acid with the highest Ka, the dissociation constant for the acid, will have the
highest H+ concentration and the lowest pH. According to the information in Table 1,
HNO2 has the highest Ka. Thus, A is the best answer.

B
) 0.1 M CH3COOH(aq)

C
) 0.1 M HClO(aq)

D
) 0.1 M NH4NO3(aq)

Solution

Guess
40. Which of the following mixtures, with each component present at a
concentration of 0.1 M, has a pH closest to 7?
A
) HClO(aq) and NaClO(aq)

When the acid and conjugate base concentrations are equal, the pH of a buffer
equals the pKa of the acid. The pKa’s of the acids can be estimated on the basis of
the Ka values in Table 1. The pKa of HClO is closest to 7. Thus, A is the best answer.

B
) HNO2(aq) and NaNO2(aq)

C
) CH3COOH(aq) and NaCH3COO(aq)

D
) HNO3(aq) and NaNO3(aq)

Solution

Guess
41. Which of the following equations correctly represents the dissolution of
NH4NO3(s) in water?

A
) NH4NO3(s)NH4(aq) + NO3(aq)

B
) NH4NO3(s)NH4–(aq) + NO3+(aq)

C
) NH4NO3(s)NH2+(aq) + NO2–(aq) + H2O

D
) NH4NO3(s)NH4+(aq) + NO3–(aq)

Upon dissolution in water, NH4NO3(s) will dissociate, forming NH4+ ions and NO3–
ions. Thus, D is the best answer.

Solution

Guess
42. When 2.0 mL of 0.1 M NaOH(aq) is added to 100 mL of a solution containing
0.1 M HClO(aq) and 0.1 M NaClO(aq), what type of change in the pH of the solution
takes place?

A
) A slight (<0.1 pH unit) increase
The pH will increase because a strong base is added. The increase will be small
because the base is being added to a buffer solution. Thus, A is the best answer.

B
) A slight (<0.1 pH unit) decrease

C
) A significant (>1.0 pH unit) increase

D
) A significant (>1.0 pH unit) decrease

Solution

Guess
43. What is the best explanation for the fact that a solution of NaNO2(aq) is basic?

A
) NO2– is hydrolyzed with the formation of OH –(aq) ions.

The NO2- reacts with water, forming OH- ions. Thus, A is the best answer.

B
) Na+ is hydrolyzed with the formation of OH –(aq) ions.

C
) NaNO2(aq) decreases the Ka of HNO2(aq).

D
) NaNO2(aq) increases the Ka of HNO2

Solution

Guess
44. Suppose that the atmospheric pressure suddenly dropped at one end of a
large lake, inducing a seiche like that shown in Figure 1. The atmospheric pressure
differential between the two ends of a lake is directly proportional to the:

A
) frequency of the oscillations.

B
) period of the oscillations.

C
) wave speed.

D
) amplitude of the oscillations.

A and B are ruled out because they are inverses of each other and depend only on L,
g, and H. C is ruled out because wave velocity is a property of the water only. D is
logical because the pressure drop at one end of the lake will cause a change in the
water’s equilibrium level there, and this change is directly related to a seiche’s
amplitude. Thus, D is the best answer.

Solution

Guess
45. The principal restoring force responsible for maintaining the sloshing
oscillations in a body of “dry water” for which surface tension is very small is the:

A
) gravitational force.

The gravitational force will drive the water’s movement, much as a pendulum moves
under the influence of gravity. That is why g is explicitly in the equation for frequency.
In addition, B, C, and D are not forces. Thus, A is the best answer.
B
) viscosity of the water.

C
) atmospheric pressure above the water.

D
) hydrostatic pressure at the bottom of the container.

Solution

Guess
46. Compute the period of oscillation for the fundamental mode of a seiche
induced in a lake that averages a depth of 30 m, with a length of 6000 m over which
the wave propagates.

A
) 50π s

B
) 200π s

The period of oscillation T is

Thus, B is the best answer.

C
) 300π s
D
) 400π s

Solution

Guess
47. The actual oscillating surface in Figure 1 would not remain precisely flat; it
would have a half-sine-wave shape. Use this fact to determine the wavespeed v of
the fundamental mode of oscillation.

A
) v = (2gH)1/2

B
) v = (3gH)1/2

C
) v = (3gH)1/2/π

D
) v = 2(3gH)1/2/π

The wave speed v is given by the product of f and the wavelength λ. Because L is
one-half of a wavelength (just like a standing wave in a tube of length L at its
fundamental vibration), v will be

Thus, D is the best answer.


Solution

Guess
48. It was argued in the passage that certain simplifying assumptions led to the
equation for frequency, which gives frequencies no more than 10% different from the
observed seiche frequencies. This equation would prove to be greatly in error,
though, for a:

A
) container with a large width L.

B
) location where the acceleration of gravity is not 10 m/s2.

C
) liquid that is very viscous.

The passage explicitly states that the frequency equation given in the passage is only
good if the viscosity is ignored, which cannot be done in C. The conditions in the
other options will not affect the validity of the approximation. Thus, C is the best
answer.

D
) liquid with zero surface tension.
Solution

Guess
49. Assume that a pan of “dry water” is momentarily disturbed. Which of the
following concepts best explains why the resulting sloshing oscillations persist for a
fairly long time?

A
) Energy conservation

Although some friction (energy loss) will take place, energy will be very nearly
conserved over each oscillation period. As a result, the amplitude of the seiches will
decay very slowly as mechanical energy is turned into heat. The other options do not
pertain to the sloshing oscillations. Thus, A is the best answer.

B
) Momentum conservation

C
) Newton’s third law

D
) Archimedes’ principle

Solution

Guess
50. Regarding Figure 1, which velocity profile depicted below best shows the
variation in velocities across the air–liquid interface of II just after I has occurred?

A
)
B
)

Because the speed must be greatest at the ends and zero in the middle, A and D are
eliminated. Furthermore, because the velocity must point down on the left and up on
the right in order to form the waveform of Part II from that of Part I, C is also
eliminated. Thus, B is the best answer.

C
)

D
)

Solution
Guess
51. In which of the following does sound travel most rapidly?

A
) Air (0°C)

B
) Water (10°C)

C
) Iron (20°C)

The speed of sound is greatest by far in a solid because of the strong intermolecular
bonds and close proximity of the molecules. Because the temperatures are close to
each other (and even favor iron slightly), iron will have the highest sound speed.
Thus, C is the best answer.
D
) Sound travels at approximately the same speed in all of the above.

Solution

Guess
52. When beryllium (Be) is bombarded with deuterons (H) of 10 MeV energy, a
deuteron is absorbed and a neutron is emitted. Which of the following is formed?

A
) Li

B
) Be

C
) B

The net result of absorbing a deuteron and emitting a neutron is for a nucleus to gain
one proton and no neutrons, so a 9Be nucleus will end up as a 10B nucleus. Thus, C is
the best answer.

D
) B

Solution

Guess
53.

Which action involves more work: lifting a weight from A to B or lowering the weight
from B to A?

A
) Lifting from A to B

B
) Lowering from B to A
C
) Equal work in both actions

The work done is mgΔh, and because none of these values changes in magnitude
when the mass goes up or down, these actions involve the same amount of work.
Thus, C is the best answer.

D
) No work is required using a pulley.

Solution

Guess
54.

What is the standard emf for the galvanic cell in which the following overall reaction
occurs?

2Na(s) + Cl2(g) ® 2Na+(aq) + 2Cl–(aq)

Half-reaction
E° red
(V)

Na+(aq) + e– ® Na(s)

–2.71

Cl2(g) + 2e– ® 2Cl–(aq)

+1.36

A
) –1.35 V

B
) +1.35 V

C
) +4.07 V

According to the table, the oxidation of Na has a potential of +2.71 V and the
reduction of Cl2 has a potential of +1.36 V. The standard emf for the galvanic cell is
therefore +2.71 V + 1.36 V = +4.07 V. Thus, C is the best answer.
D
) +6.78 V
Solution

Guess
55. Which of the following shows the electron configuration of chlorine in NaCl?
A
) 1s22s22p63s23p4

B
) 1s22s22p63s23p5

C
) 1s22s22p63s23p6

In NaCl, chlorine exists as the chloride ion. A chloride ion has 18 electrons with the
electron configuration 1s22s22p63s23p6. Thus, C is the best answer.
D
) 1s22s22p63s23p44s

Solution

Guess
56. Which of the following statements could explain the frequently bluish color of
EQLs?

A
) Sodium salts are common in the earth’s crust, and sodium emissions
can be quite bright.

B
) In transparent substances, dispersion effects are in general greater for
longer wavelengths.

C
) The ultraviolet radiation is absorbed by molecules that then fluoresce
at yet shorter wavelengths.

D
) The ultraviolet radiation is absorbed by molecules that then fluoresce
at yet longer wavelengths.

D is physically possible (although fluorescence does not limit the light to being blue),
and so it could explain the bluish color of EQLs. C cannot occur (energy is not
conserved), and options A (sodium emits yellow, not blue) and B (shorter
wavelengths, not longer, are more affected) are not reasonable. Thus, D is the best
answer.

Solution

Guess
57. During their compression, little heat is lost by conduction from the hot vapor
bubbles responsible for SL effects because:

A
) the process occurs too rapidly for heat loss to be appreciable.

Rapid processes are adiabatic, leaving little time for heat to be lost. B and C do not
make physical sense. D may be true but is not relevant to the question. Thus, A is the
best answer.
B
) the heat is carried on the advancing wavefront.

C
) the surrounding liquid is subjected to the same compressional force.

D
) convection predominates over other processes in liquids at ordinary
temperatures.
Solution

Guess
58. Heating of the vapor bubbles occurs during the compression phase of the
passing waves in SL because:

A
) the heat of vaporization of water is high and serves as a barrier to the
effect.

B
) constructive interference in the wave motion is greater than at other
times.

C
) work is being done on the vapor bubbles by forces external to them at
that time.

Only C makes physical sense. Indeed, the compression of the bubble means that
work is being done on the bubbles, resulting in the vapor bubbles absorbing heat as
their volume goes down. Thus, C is the best answer.

D
) energy propagates primarily by means of transverse waves at that
time.

Solution

Guess
59. Atomic hydrogen has a characteristic spectral emission at a wavelength of
6.56 × 10–7 m that might contribute to EQLs. What color is this characteristic H
emission?

A
) Violet

B
) Blue

C
) Green

D
) Red
This requires straightforward knowledge of the visible spectrum. The visible spectrum
ranges from 400 to 700 nm, and light with wavelengths above ~620 nm is red. This
means that a wavelength of 6.56 x 10-7 m (656 nm) is in the red part of the
spectrum. Thus, D is the best answer.

Solution

Guess
60. Aluminum belongs to what block of elements in the periodic table?

A
) s

B
) p

Aluminum is found in group 13 (3A) of the periodic table; it is a p-block element.


Thus, B is the best answer.

C
) d

D
) f

Solution

Guess
61. What is the oxidation number of aluminum in Na[Al(OH)4](aq)?

A
) +1

B
) +2

C
) +3

The oxidation number of Na is +1, and the charge on a hydroxide ion is –1. For the
compound to be neutral, aluminum would have to have an oxidation number of +3.
Thus, C is the best answer.
D
) +4

Solution

Guess
62. What is the geometry of the hexafluoroaluminate ion (AlF63–)?

A
) Octahedral
An ion consisting of a central metal ion and six ligands is expected to have an
octahedral geometry. Thus, A is the best answer.

B
) Tetrahedral

C
) Trigonal bipyramidal

D
) Hexagonal
Solution

Guess
63. Approximately how much Al2O3 is required to make 100 kg of Al?

A
) 500 kg

B
) 200 kg

Equation 4 shows that 4 mol Al is produced from 2 mol Al2O3.

100,000 g Al × (1 mol Al/27 g Al) × (2 mol Al2O3/4 mol Al) × (102 g/mol Al2O3) =
188.9 kg Al2O3

Thus, B is the best answer.

C
) 80 kg

D
) 50 kg

Solution

Guess
64. In the reaction shown in Equation 1, Al(OH)3 acts as what kind of acid or
base?

A
) Lewis acid

In the reaction shown in Equation 1, the Al in Al(OH)3 accepts an electron pair from
the OH– of NaOH and is therefore acting as a Lewis acid. Thus, A is the best answer.

B
) Lewis base
C
) Brønsted acid

D
) Brønsted base

Solution

Guess
65. At which electrode is aluminum produced in a galvanic cell and in an
electrolytic cell?

A
) At the anode in both cells

B
) At the cathode in both cells

Reduction of the Al3+ to form Al(s) occurs at the cathode both in a galvanic and in an
electrolytic cell. Thus, B is the best answer.

C
) At the anode in the galvanic cell and cathode in the electrolytic cell

D
) At the cathode in the galvanic cell and anode in the electrolytic cell

Solution

Guess
66. In the reaction shown in Equation 2, three moles of Al(OH)3 is chemically
equivalent to what volume of CO2(g) measured at 1 atm and 0°C?

A
) 11.2 L

B
) 16.8 L

C
) 22.4 L

D
) 33.6 L

In Equation 2, 2 mol Al(OH)3 is produced from 1 mol CO2. It follows that 3 mol
Al(OH)3 is produced from 1.5 mol CO2. At 1 atm and 0ºC, the volume of 1.5 mol CO2
is (22.4 L / 1 mol) × 1.5 mol = 33.6 L. Thus, D is the best answer.

Solution
Guess
67. Changing which of the following will change the focal length of the convex
mirror in Figure 2?

A
) Index of refraction of the mirror

B
) Radius of curvature of the mirror

The focal length of the mirror depends only on the radius of the curvature. Because
there is no refraction, the index of refraction is irrelevant and the properties of Lens B
will not affect the focal length of the mirror. Thus, B is the best answer.

C
) Position of the lens at B

D
) Focal length of the lens at B

Solution

Guess
68. As the light passes from the air into the glass, it makes an angle qa in air and
an angle ql in the lens material, relative to the normal at the surface. What equation
relates the angles ql and qa?

A
) qa = ql

B
) 1/qa = 1/ql

C
) nasin qa = nlsin ql

This item requires knowledge of Snell’s law of refraction, which is given by C. The
other options are erroneous. Thus, C is the best answer.

D
) na/sin qa = nl/sin ql

Solution

Guess
69.

The converging lens in Figure 1 is removed and the diverging lens is placed in
position B, as shown in the figure below. Which of the following best describes the
light rays from the diverging lens in this configuration?

A
) Converging rays
B
) Parallel rays in and out

C
) Reflected rays diverging

D
) Diverging rays

Each light ray will refract at each surface of the lens and will increasingly diverge
from the others because of the shape of each surface, so only diverging rays can
result. Thus, D is the best answer.
Solution

Guess
70. If a very bright light source shines on a mirror, the mirror may become warm
because:

A
) all of the light is reflected, and, by momentum conservation, the
molecules in the mirror move, producing heat energy.

B
) some of the light passes through the mirror, and, by energy
conservation, potential energy is produced.

C
) some of the light is absorbed by the mirror, and, by energy
conservation, thermal energy is produced.

In general some of the light energy will be absorbed by the mirror, causing some
heating to take place (although the amount will be small for a high-quality optical
mirror). Thus, C is the best answer.

D
) none of the light is reflected, and, by energy conservation, mass is
converted to energy.
Solution

Guess
71. Visible light travels more slowly through an optically dense medium than
through a vacuum. A possible explanation for this could be that the light:

A
) is absorbed and re-emitted by the atomic structure of the optically
dense medium.

A is known to occur and is the reason for the slowing down of light. B is incorrect
because the nucleus is involved. C is incorrect because the motion of the photons is
certainly not random. D is true but does not answer the question. Thus, A is the best
answer.

B
) is absorbed and re-emitted by the nucleus of the material in the
optically dense medium.

C
) bounces around randomly inside of the optically dense medium before
emerging.

D
) loses amplitude as it passes through the optically dense medium.

Solution

Guess
72. What is the conjugate base of the bisulfate ion (HSO4–)?

A
) H+

B
) OH–

C
) SO42–

The conjugate base of HSO4-, formed by the loss of one proton, is SO42-. Thus, C is
the best answer.

D
) H2SO4

Solution

Guess
73. If the magnitude of a positive charge is tripled, what is the ratio of the original
value of the electric field at a point to the new value of the electric field at that same
point?

A
) 1:2

B
) 1:3

The magnitude of the electric field E of a point charge is given by

If q is tripled, E also will be tripled. Thus, B is the best answer.

C
) 1:6

D
) 1:9
Solution

Guess
74. A 7-N force and an 11-N force act on an object at the same time. Which of the
following CANNOT be the magnitude of the sum of these forces?

A
) 2N

The largest net force occurs when the forces point in the same direction; then, the
magnitudes will add mathematically to a value of 18 N. Similarly, the smallest net
force occurs when the forces point in opposite directions and the lesser magnitude is
subtracted from the greater; this results in a value of 4 N. Of the answers listed, only
A (2 N) does not fall between these values. Thus, A is the best answer.

B
) 8N

C
) 12 N

D
) 18 N

Solution

Guess
75. A student plans to add HCl to a solution containing Pb(NO3)2(aq). To
determine how much Pb2+ will precipitate from solution when the HCl is added, the
student needs to know which of the following?

A
) Ka for HCl

B
) Ka for HNO3

C
) Ksp for PbCl2

To determine how much Pb2+ will precipitate, the student must know the solubility of
PbCl2, which is related to the solubility product constant Ksp of PbCl2. Thus, C is the
best answer.

D
) Keq for the reaction Pb2+ + 2 e– Pb

Solution

Guess
76.
A block of weight W is pulled across a rough floor by a rope that exerts a force T on
the block. The frictional force between the floor and the block is F. Which of the
following expressions equals the frictional force F when the block moves with a
constant speed?

A
) T

B
) W–T

C
) T sin q

D
) T cos q

Because the object moves at a constant speed, the net force on the object is zero. In
the horizontal direction the sum of the forces must be zero. This will occur if the
horizontal component of T, T cos θ, has the same magnitude as the frictional force F.
Thus, D is the best answer.
Solution

Guess
77. When an element undergoes β decay, a nuclear neutron is converted to a
nuclear proton as the nucleus emits an electron. What happens to the atomic number
and atomic mass of an element that undergoes β decay?

A
) The atomic number increases, but the atomic mass stays
approximately the same.

An electron has a negative charge and very little mass compared to the charge and
mass of nucleons. If an electron is emitted by a nucleus to balance the charge, the
nucleus must gain one positive charge (a proton); thus, the atomic number (number
of protons in nucleus) increases by one. However, the atomic mass is nearly
unchanged because an electron has only 0.05% of a proton’s mass. Thus, A is the
best answer.

B
) The atomic number stays the same, but the atomic mass decreases.

C
) Both the atomic number and the atomic mass decrease.

D
) The atomic number decreases, but the atomic mass stays
approximately the same.

78. The passage discussion of male and female experience assumes that:
B
) there is a degree of similarity between female and male experience.

The following statement emphasizes that there is a degree of similarity between


male and female experience, even in the face of differences: “That such experiences
[male and female] differ, there can be no disagreement; but that such experiences
overlap, there should also be no disagreement”.
79.
A
) portray women struggling to achieve identity in a patriarchy.

The main theme of the passage is the struggle of women for identity in a patriarchy:
“There is by now a sizable body of fiction that focuses on female experiences or
conditions, in which women must find their way personally, professionally, socially, in
what is basically a patriarchy. This term we may define as any society in which men
control authority and determine the roles women should or should not play”. This
theme is augmented by the passage discussion of how Bertha in Jane Eyre represents
“the characteristic victim of a male-dominated society”. It is further augmented by
the passage author’s reference to Virginia Woolfe’s discussion of how certain of the
older generic and stylistic forms of writing are unsuited to women and how the novel
was new enough to be amenable to women’s use.

Solution

Guess
80. The author asserts that the novelist Jean Rhys:

A
) reworked the character of Bertha in a way no male writer could have.

The passage author reinforces this point with the following comment: “There is, I feel,
no male novelist who could have picked up the thread of Bertha’s existence and
turned it into an emblem, as Jean Rhys did; and here alone we note the way the
female novelist can perceive aspects of experience that remain (at least in our era)
outside the reach of the male writer. Reading back from Rhys, we experience Jane
Eyre differently”.

Solution

Guess
81. The author suggests that Bertha’s imprisonment in the upper reaches of
Thornfield:

1. could have been explored equally well by male or female novelists.


2. provided Jean Rhys with an archetypal symbol of the plight of women.
3. functioned for Charlotte Brontë primarily as a plot device.
D
) II and III only

The passage author comments that, according to Rhys’s view, “Bertha’s plight is
more archetypically female than Jane’s, by far, because Jane is moving in a fairy tale
of sorts in which elements yield to her, whereas Bertha has moved in the real world
of power”. Bertha also functions as a plot device and “is presented as the element
that must be eliminated in order for Rochester and Jane to complete their destiny
together”. The passage author goes on to suggest that this was determined by the
genre: “Because Charlotte Bronte was writing a romance, Bertha could become
expendable”.
82. According to the passage, Virginia Woolf believed the novel was more suitable
to women writers than was the epic or the poetic play because the:

A
) novel was a more recent and thus more flexible genre.

The passage author refers to Virginia Woolfe’s discussion in A Room of One’s Own of
the suitability for women of certain sentences written by men along with older
literary forms such as the epic and poetic play. The author quotes Virginia Woolfe as
follows: “[A]ll the older forms of literature were hardened and set by the time she
became a writer. The novel alone was young enough to be soft in her hands”.

83. In order for the process described in paragraph 3 to occur, the


temperature of ice crystals in a cloud must be higher than that of:
D
) the supercooled droplets.

The author notes that ice crystals evaporate the supercooled droplet; therefore, it
logically follows that the ice crystals would need to be warmer for this process to
occur.

84. According to the passage, a cloud is formed when:


C
) humidity turns a large number of dust particles into water droplets.

High relative humidity is a key factor in cloud formation: “When the relative
humidity is high, water vapor condenses on many particles and begins to form
droplets. . . . An actual cloud materializes when humidity reaches a certain critical
value which turns most of the dust particles into water droplets”.

85. On the sole basis of the passage, one could conclude that it might be
possible to reduce the rainfall in a region by:
B
) decreasing the number of particles in the air.

The amount of water condensing on dust particles in the air and forming clouds
accounts for rain, according to the passage: “An actual cloud materializes when
humidity reaches a certain critical value which turns most of the dust particles
into water droplets. Under the right conditions, the cloud droplets combine rapidly
into raindrops. . . .”. Another theory is that large dust nuclei sweep up the smaller
ones and grow into the size of a raindrop: “A cloud will produce rain, according to
this theory, when it contains sufficient moisture and a suitable number of giant
nuclei”. According to both theories, it logically follows that a decreased number of
particles in the air would decrease the chances for rain.

86. The passage assertion that salt is largely responsible for rainfall from warm
clouds is based on evidence that:

1. salt particles are spread throughout the atmosphere.


2. the amount of salt in rainfall is related to the amount of salt carried inland from
the sea.
3. the number of drops per unit volume in rain over the sea is similar to that of salt
particles in ocean air.
D
) I, II, and III
The passage indicates that all three of these conditions exist: salt particles are
spread throughout the atmosphere; “there is a statistical evidence of a
relationship between the amount of salt carried inland from the sea and the
amount of salt in our rainfall”; and a finding indicates that “the number of drops
per unit volume in rain over the sea is about the same as the number of salt
particles in the ocean air”.

86. Which of the following statements most strongly challenges the author’s
assertions about the way raindrops are formed in clouds at subfreezing
temperatures?

C
) No ice crystals are present in the upper regions of clouds at high
altitudes.

According to one theory proposed by the author, if ice crystals are present, they
absorb the supercooled water droplets, allowing them to grow to a large size and fall
as either snow or rain. Because ice crystals are a key to the process described by this
theory, an absence of ice crystals in the upper regions of clouds at high altitudes
would seriously challenge this theory.

88. Assume that a particular inland region in a warm climate receives a great deal
of rain. Given the information in the passage, which of the following proposed
explanations of this phenomenon is the LEAST plausible?

A
) There is very little wind over the region.

According to the passage, salt particles act as nuclei that are instrumental in the
formation of raindrops and precipitation. Wind is indicated in the passage as being
instrumental in carrying salt particles over land. Therefore, a lack of wind over the
region makes it far less likely that the rain-producing salt particles would be carried
inland.

Solution

Guess
89. The passage suggests that the example of an eighteenth-century
American businessperson who succeeded in a competitive market but did
NOT fit comfortably into a self-governing society would best support the
view of:
B
) de Tocqueville.

In commenting about how Crèvecoeur failed to see “that a purely economic


person would be as unsuited to a self-governing society as would the rank-bound
subject of traditional regimes”, the author suggests that de Tocqueville “gave a
much more adequate view”. The author later specifies how de Tocqueville viewed
the purely economic man by commenting that de Tocqueville “appreciated the
commercial and entrepreneurial spirit [in the U.S.] . . . but saw it as having
ambiguous and problematic implications for the future of American freedom”.

90. According to the passage, de Tocqueville thought that the key to America’s
success as a republic was its:
D
) citizens’ moral and intellectual dispositions.

The author points out that de Tocqueville believed that mores (moeurs), the “habits
of the heart,” shaped the mental habits and the moral and intellectual disposition
that contributed the most to the American character.

91. The passage suggests that de Tocqueville’s analysis of America might have
been more accurate than Crèvecoeur’s because de Tocqueville possessed:
B
) a belief in republican principles.

De Tocqueville’s belief in republican principles is underscored by the author’s


discussion of how mores, more than even laws or physical circumstances, contributed
to maintaining the American democratic republic. The author further maintains that
this is a fuller, more “adequate view” than Crèvecoeur’s: “[De Tocqueville] added to
Crèvecoeur’s earlier sketch a more penetrating and complex understanding of the
new society, informed by republican convictions and a deep sensitivity to the place of
religion in human life”.

91. According to passage information, which of the following factors would be


the best example of the “mores” of which de Tocqueville speaks in
Democracy in America?
C
) Religious faith

The author notes that de Tocqueville was informed by “a deep sensitivity to the
place of religion in human life”. It may then be reasonable to conclude that
religious faith may be one of the factors informing the mores that de Tocqueville
saw as shaping American life and maintaining it as a democratic republic.

93. According to the passage, the optimism of French Enlightenment scholars was
called into question by:

1. the economic development of Americans.


2. the French Revolution and its aftermath.
3. conditions in English mill towns.

C
) II and III only

In discussing de Tocqueville’s views, the author is explicit on this point: “For de


Tocqueville, the optimism of the Enlightenment had been tempered by the
experience of the French Revolution and its aftermath, and the prophecies of the
early political economists were finding an alarmingly negative fulfillment in the
industrial infernos of English mill towns”.

94. On the basis of the passage, a comparison of Crèvecoeur’s and de


Tocqueville’s views of America might reasonably lead to the conclusion
that:
B
) de Tocqueville was somewhat more pessimistic than Crèvecoeur about
America’s future.
Crèvecoeur based his optimism on the entrepreneurial spirit and self-made, self-
reliant qualities of the American “Economic Being,” which the author calls “an
emancipated, enlightened individual confidently directing energies toward the
environment, both natural and social, aiming to wring from it a comfortable
happiness”. However, the author points out that Crèvecoeur could not see what
many Americans of his time saw, namely, “that a purely economic person would be
unsuited to a self-governing society. . . .”. De Tocqueville found reason for pessimism
where Crèvecoeur found reason for optimism: “[De Tocqueville] appreciated the
commercial and entrepreneurial spirit that Crèvecoeur had emphasized but saw it as
having ambiguous and problematic implications for the future of American freedom”.

95. Which of the following conclusions can justifiably be drawn from the passage?

1. Crèvecoeur believed that economic advancement was linked to individual self-


interest.
2. De Tocqueville believed that undermining American mores would threaten the
country’s free institutions.
3. Crèvecoeur and de Tocqueville both predicted a swift demise for the American
experiment.

C
) I and II only

The author comments that Crèvecoeur saw economic well-being as timed to the
rational, self-interested individual. The author quotes Crèvecoeur as follows: “ ‘Here
the rewards of [the American’s] industry follow with equal steps the progress of his
labour; his labour is founded on the basis of nature, self-interest; can it want a
stronger allurement’”? The author states that de Tocqueville stressed how American
mores “had been the key to the Americans’ success . . . and that undermining
American mores was the most certain road to undermining the free institutions of the
United States”.

96. According to the passage, the reason the La Mojarra writing system is thought
to have been more phonetic than the Mayan system is that:

A
) fewer glyphs were used in the La Mojarra writing system.

The author points out: “The La Mojarra (Olmec) language may have fewer signs than
Maya and may thus represent an even more phonetic, less logographic system”.

97. Is Schele’s interpretation of Mayan concerns directly supported by


evidence presented in the passage?
D
) No; it is not explicitly supported by passage information.

The author indicates that monument carving ceased before these concerns arose,
thus leaving behind no revealing record. Therefore, Schele’s reasons remain
speculative.

98. According to passage information, which of the following factors


distinguishes the Maya from the Aztecs?
B
) The Maya probably created their own writing system.
Passage information makes no indication that the Aztecs developed their own
writing system whereas the author indicates that inscriptions on monuments
provided evidence for the similarities between Mayan and Aztec civilizations as
bloody and warlike societies. In this sense, the passage makes a distinction
between the Aztecs and the Maya by referring only to the Maya as having had a
writing system.

99. According to the passage, the late-Classic Mayan collapse probably


occurred during which of the following time periods?
D
) After A.D. 900

The passage notes: “Because monument carving ceased, as far as we know, after
A.D. 909, we may never have a revealing record of [the collapse of Mayan
civilization]. . . ”. Given this information, it logically follows that the collapse of
Mayan civilization would have occurred sometime after A.D. 900 or, more
specifically, after A.D. 909.

100. Which of the following statements, if true, would best support the view that
Mayan civilization was peaceful and contemplative, given the recent translations of
Mayan glyphs discussed in the passage?

A
) Mayan depictions of warfare and violent rites were purely symbolic.

The fact that the depiction of warfare and bloody rites was purely symbolic would
indicate that in actual practice Mayan civilization found less violent means of
carrying out rituals, maintaining authority, and resolving feuds.

100. The central argument of the passage is that the agrarian myth:
B
) is a sentimental representation of the role that agriculture played in
American life.

The author discusses how the agrarian myth reflects a sentimental notion of
American rural life: “Americans were raised with a sentimental attachment to
rural living [that] represents a kind of homage that Americans have paid to the
fancied innocence of their origins”. The author also notes that this sentimental
view relates to a conception of the yeoman farmer as a kind of hero who
represents the ideal man and ideal citizen. This myth appeared to Americans “as
the victory of a band of embattled farmers over an empire . . . [that] seemed to
confirm the moral and civic superiority of the yeoman, made the farmer a symbol
of the new nation, and wove the agrarian myth into its patriotic sentiments and
republican idealism”.

102. The passage suggests that the agrarian myth originated:

A
) in literature.

The author points out: “In origin the agrarian myth was not a popular but a
literary idea . . .”.

103. Based on the passage, the agrarian myth assumes that:


1. yeoman farmers are wholesome and honest.
2. yeoman farmers are morally superior to most citizens.
3. agriculture deserves special treatment from the government.
D
) I, II, and III

In describing the yeoman, the author comments: “The yeoman . . . was the
incarnation of the simple, honest, independent, healthy, happy human being.
Because he lived in close communion with beneficent nature, his life was believed to
have a wholesomeness and integrity impossible for the depraved populations of
cities”. The author discusses how the perception that the American Revolution
represented “the victory of a band of embattled farmers over an empire, seemed to
confirm the moral and civic superiority of the yeoman, made the farmer a symbol of
the new nation, and wove the agrarian myth into its patriotic sentiments and
republican idealism”. The author also points out that these qualities conferred
special status and consideration upon the yeoman farmer: “Unstinted praise of the
special virtues of the farmer and the special values of rural life was coupled with the
assertion that agriculture, as a calling uniquely productive and uniquely important to
society, had a special right to the concern and protection of government”.

Solution

104. Based on the passage, the agrarian myth became part of a mass creed
because:
B
) farmers were credited with having played a major role in the American
victory in the Revolutionary War.

In discussing how the agrarian myth shifted from the upper classes to become part of
the “mass creed” in the early nineteenth century, the author comments: “The roots
of this change may be found as far back as the American Revolution which,
appearing to many Americans as the victory of a band of embattled farmers over an
empire, seemed to confirm the moral and civic superiority of the yeoman, made the
farmer a symbol of the new nation, and wove the agrarian myth into its patriotic
sentiments and republican idealism”.

105. According to the passage, the agrarian myth implied that yeoman
farmers were:

D
) happy and industrious.

The author attributes the happiness of the yeoman farmer to his wholesome way of
life: “The yeoman . . . was the incarnation of the simple, honest, independent,
healthy, happy human being. Because he lived in close communion with beneficent
nature, his life was believed to have a wholesomeness and integrity impossible for
the depraved populations of cities”. The author also discusses how the productivity
of the farmer entitled agriculture to a privileged position in society worthy of special
consideration from the government. Even when independent farmers turned to
commercial farming in the nineteenth century, “[t]hey were a hardworking lot in the
old tradition”. By “old tradition,” the author means the independent yeoman farmer
of the eighteenth century
106. Which of the following assertions, if true, would most weaken the main
point of the passage?

C
) The agrarian myth played a part in the thinking of only a handful of
Americans during the eighteenth and nineteenth centuries.

This information contradicts the author’s assertion that the agrarian myth “was
clearly formulated and almost universally accepted in America during the last half of
the eighteenth century”.

107. What does the passage suggest about whether or not the agrarian
myth was false?

D
) Its stereotypes corresponded to many of the realities of early American
agricultural life.

The author points out that in the eighteenth century there was some truth to the
agrarian myth: “When [the agrarian myth] took form in America during the
eighteenth century, its stereotypes did indeed correspond to many of the realities of
American agricultural life. There were commercial elements in colonial agriculture
almost from the earliest days, but there were also large numbers of the kind of
independent yeoman idealized in the myth”.

108. As it is used in the first sentence, sentimental most nearly means:

B
) emotional.
Sentimental is used here to describe the implicit emotion associated with the
idealization of the yeoman farmer in the agrarian myth: “The yeoman . . . was the
incarnation of the simple, honest, independent, healthy, happy human being.
Because he lived in close communion with beneficent nature, his life was believed to
have a wholesomeness and integrity impossible for the depraved populations of
cities”. The author explains how the agrarian myth finds expression in nationalist
ideology and how this is tied to patriotic feeling. This feeling traces “as far back as
the American Revolution, which, appearing to many Americans as the victory of a
band of embattled farmers over an empire, seemed to confirm the moral and civic
superiority of the yeoman, made the farmer a symbol of the new nation, and wove
the agrarian myth into its patriotic sentiments and republican idealism”.

109. According to the passage, what was the effect of the triumph of commercial
agriculture?

A
) It largely removed those conditions that had given the agrarian myth
much of its original force.

The author states this directly, while also pointing out that the agrarian ideal was
replaced by an even more potent ideal: “The triumph of commercial agriculture not
only rendered obsolete the objective conditions that had given to the agrarian myth
so much of its original force, but also showed that the ideal implicit in the myth was
contesting the ground with another, even stronger ideal—the notion of opportunity,
of career, of the self-made man”.

110. The passage suggests that the same forces that encouraged the
equalitarian theme in the agrarian myth led to the:

C
) emergence of an unprecedented entrepreneurial zeal.
The author states: “The same forces in American life that had given to the
equalitarian theme in the agrarian romance its most compelling appeal had also
unleashed in the nation an entrepreneurial zeal probably without precedent in
history, a rage for business, for profits, for opportunity, for advancement” (lines 70–
75).

111. Which of the following statements best summarizes the central problem
addressed by the passage?
C
) Unbiased thought is inconsistent with human survival.

This is a theme repeated throughout the passage. The author notes: “Sensibility
and intelligence arose in . . . [humans] to serve animal purposes, for through the
first, it was able to distinguish those things that favor the survival of it and its
race, and through the second, it was able to go about in a more efficient manner
to secure them”. The author then notes the consequences of intellectual
detachment: “When sensibility has been detached from its animal setting, it may
develop into a quest for that self-justifying beauty which is humanly valuable but
biologically useless. When intelligence is detached, it not only tends to paralyze
natural impulse by criticizing natural aims but develops certain intellectual virtues
which are biological vices”.

112. Which of the following statements, if true, would most directly undermine
the author’s central argument?
C
) Nonhuman animals often behave altruistically.

The author seems to privilege aggression as the means of ensuring survival and
argues that calling this into question causes civilizations founded on aggression to
crumble. The author criticizes the development of intellectual virtues which become
biological vices. Presumably, these intellectual virtues and biological vices include
altruism—and the author does later point this out: “Civilizations die from . . . the
sense of fair play quite as surely as they die of debauchery”.

113. Suppose that persons of average intelligence tend to have higher incomes
than those of very intelligent persons. The author would be most likely to
argue that this difference exists because:

D
) a highly developed intelligence inhibits competitive action.

The author views higher or “detached” intelligence as a liability, placing people at a


disadvantage: “It puts its possessor at a disadvantage in dealing with those whose
intelligence faithfully serves their purpose by enabling them to scheme for their ends
and to justify to themselves their desires. Such is the animal function of intelligence,
and whenever it develops beyond this level, it inhibits rather than aids that effective
action in the pursuit of natural ends which was the original function of the mind”.
114. Which of the following passage contentions might it be possible to refute by
clear counterexamples?

B
) III only

Concrete evidence refuting this statement could be found by research into the
historical record, which could potentially reveal that factors other than aggression
were instrumental in the founding of great nations.

115. Some research into unconscious motivation suggests that even apparently
impartial thought processes may be deeply self-serving. What is the relevance of this
consideration to the author’s argument?

A
) It weakens the distinction drawn between “animal” and “human” uses
of intelligence.

One of the key passage arguments is that impartial thought is characteristically


human and separates human intelligence from its self-serving animal function.
The finding that even impartial thought processes are self-serving blurs perhaps
even eliminates this distinction.
116. Which of the following research findings would undermine the
argument about the development of color-word synesthesia?
B
) Color-word synesthesia can develop in literate adults.

According to the passage argument, color-word synesthesia develops when a


child is learning to write and, thus, “emerges during a critical period of maximum
plasticity in the visual system, when it is involved in learning to link letters with
sounds and strings of letters with objects”. A finding that synesthesia can
emerge in a literate adult would undermine the argument that synesthesia
develops during a period of rapid learning and development, that is, maximum
plasticity, in children.

117. According to the author’s explanation, one pair of words that would be
likely to evoke the same synesthetic experience is:
B
) knit and kit.

The passage reports that the synesthetic link is between the visual appearance of
the first letter of the word and the color: “Thus, a subject reporting the perception
of red on hearing the word photograph would also report perceiving red on
hearing the word palladium but a different color on hearing the word fish”.
Accordingly, knit and kit would evoke the same synesthetic experience because
both words have the same first letter.

118. The author implies that visual synesthesia can occur because certain neurons
respond to the co-occurrence of a particular color and shape. A plausible hypothesis
is that such cells evolved because they increased processing speed in the
identification of:

A
) dangerous predators on the basis of incomplete visual information.

In the absence of a positive identification of a dangerous predator, it logically


follows that the mind would need to be able to associate a certain color with a
certain shape as a warning of the probable presence of a predator. Lacking this
ability, a person would not likely respond to incomplete information, but only to a
positive identification, which in the case of a predator could be too late.

119. As the word is used in the passage, a grapheme is best described as:
D
) one of the units of a written word.

The word as it is used here follows a discussion of how the synesthetic link “is not
between meaning and color, or sound and color, but between the visual appearance
of the first letter of the word and color”. The example given is that the words
photograph and palladium would evoke the same synesthetic experience whereas
fish and photograph would not. Therefore, it can be inferred from this discussion that
the word grapheme refers to one of the units of a written word, namely, the visual
element of letters.

120. Which of the following phenomena is an example of synesthesia?


B
) Hearing a loud note when seeing the word trumpet

As explained in the passage, synesthesia is distinguished by the ability of one


sense experience to evoke the perception of another sense experience. In this
example, the word trumpet evokes the actual perception of a loud note.

121. Which of the following conclusions about Latin is a logical inference from the
passage?
C
) Et can mean both even and too.

This inference can be made on the basis that the Latin phrase Et in Arcadia ego
was grammatically translated as “Even in Arcady, there am I,” and that the
translation was changed in the case of Poussin’s Louvre painting to “I, too, lived in
Arcady.” The author comments: “Thus Poussin himself, while making no verbal
change in the inscription, invites, almost compels, the beholder to mistranslate it
by relating the ego to a dead person and by connecting et with ego instead of
with Arcadia”. When et is connected with ego in the later mistranslation, it
becomes too, and when it is connected with Arcadia it becomes even.

122. Suppose that a painting contained words with no apparent relevance to the
scene depicted. The passage suggests that in discussing this painting, the
passage author would be most likely to:
C
) interpret the words on the basis of the scene.

When Poussin removed the death’s-head and retained the Latin phrase Et in Arcadia
ego, the words originally attributed to the death’s-head, the viewer is forced to
attribute the words to the tomb or the dead person inside the tomb and to
mistranslate the Latin phrase; thus, the viewer is forced to interpret the words on the
basis of the scene.
123. By the end of the eighteenth century, the inscription on Poussin’s second
Arcadia painting was translated as “Even in Arcady, there am I” only in
England. In conjunction with passage information, this fact most strongly
implies that in comparison with other Europeans, the English were:
D
) more influenced by the Guercino painting.

Retention of the grammatical translation would indicate that the English were more
influenced by the Guercino painting because the translation relates directly to it:
“When read according to the rules of Latin grammar (‘Even in Arcady, there am I’),
the phrase had been consistent and easily intelligible as long as the words could be
attributed to a death’s-head and as long as the shepherds were suddenly and
frighteningly interrupted in their walk. These conditions are manifestly true of
Guercino’s painting . . .”.

124. Suppose that when Poussin’s Louvre painting is cleaned, a skull is


discovered on the tomb. This discovery means that the author’s thesis about this
painting:
C
) is less plausible.

The author’s thesis would be less plausible because the viewer would then have
to reconsider the possibility that the Latin phrase could be attributed to the
death’s-head. However, the thesis would still retain some plausibility because the
shepherds in the later painting would still be engaged in calm discussion and
pensive contemplation, rather than directly engaged with or accosted by the
death’s-head as in the earlier paintings. This would hold out the possibility that
the role of the death’s-head has been demoted from an active player in the
painting to a formalized or stylized decorative object, but one that the viewer has
to consider in relation to the inscription.

125. According to the author, which details of Poussin’s Louvre painting support the
belief that it reveals his decision to reject the moralizing tradition in art?

1. A classical tomb
2. A pagan river god
3. A symmetrical composition

B
) III only

The break with the medieval moralizing tradition, as described in the passage, can be
seen in how the symmetrical arrangement of the group of shepherds on either side of
the tomb eliminates the more dramatic portrayal of the shepherds entering from the
left and being unexpectedly accosted by the death’s-head: “Instead of being checked
in their progress by an unexpected and terrifying phenomenon, they are absorbed in
calm discussion and pensive contemplation”.

126. What is the significance to the passage argument of the information that the
shepherds are already at the tomb rather than approaching it?

A
) It shows that they are not surprised by the reminder of death.
The author describes the Louvre painting as follows: “Instead of two or three
Arcadians approaching from the left in a group, we have four, symmetrically arranged
on either side of a sepulchral monument. Instead of being checked in their progress
by an unexpected and terrifying phenomenon, they are absorbed in calm discussion
and pensive contemplation”. The author later comments that they are engaged in
contemplation of mortality rather than being surprised in a dramatic encounter with
the death’s-head.

127. Which of the following statements, if true, would most weaken the
author’s reasoning about the historical significance of the changes
introduced in Poussin’s second Arcadia painting?
D
) Many of Poussin’s later paintings have strongly moralistic themes.

If this were the case, the removal of the moralistic elements from the Poussin’s
Louvre painting would be seen as an anomaly, not a major historical trend-setting
change in Poussin’s work or, as the author puts it, “a radical break with the medival,
moralizing tradition”. In other words, this “radical break,” if it simply represented an
anomaly rather than a major shift, would have less historical significance than the
author contends.

128. The author’s discussion accords best with the idea that paintings:

A
) convey ideas through the objects they represent.

The author’s discussion centers on how various key objects in the painting convey
important ideas and influence the interpretation of the painting, especially how
the removal of the death’s-head in the two Poussin paintings led to a new
interpretation of the Latin inscription Et in Arcadia ego.

129. As used in the passage, the term elegiac is closest in meaning to:

B
) serenely reflective.

The Arcadians in the second Poussin painting are characterized as being “absorbed in
calm discussion and pensive contemplation” and engaged in “a contemplative
absorption in the idea of mortality”. These descriptions accord with the meaning of
elegiac as a state of “serenely reflective” mourning.

130. What can be inferred from the passage about Guercino’s Et in


Arcadia ego painting?
B
) It encourages a grammatical translation of the phrase.

Passage information emphasizes the elements retained from the Guercino painting in
Poussin’s first painting. The author then goes on to discuss how the first painting
invites a grammatical translation of the Latin phrase Et in Arcadia ego. Therefore, it
can be reasonably inferred that the Guercino painting similarly invites a grammatical
translation because of the similarities between it and the first Poussin painting.

131. The author apparently thinks that society today plans for the future
by:
D
) extending the present situation through piecemeal innovations.

The author states: “. . . [T]he tendency during recent decades has been to limit
planning to the here and now. The future is imagined not as a really new venture but
as a mere extension of the past”. This comment reinforces an earlier observation that
what are called “advances” are simply conditioned social reflexes: “We have trained
our social reflexes for technological ‘advances,’ however trivial their goals and
deleterious their long-range effects. . . . [T]he word progress now means just moving
on, even though the forward motion is on a road that leads to disaster or despair”.

132. Which of the following statements, if true, would most weaken the author’s
argument about the way society should plan for the future?

A
) Having a goal firmly in mind decreases the chances of achieving that
goal.

This statement, if true would most weaken the author’s argument because the
author not only argues for having a goal in mind but having multiple goals, that
is, “multiple models of possible futures” and that these goals must “imply value
judgments as to what is desirable or not, good or bad, and thus inevitably give
direction to the social and scientific enterprise”.

132. Which of the following factors is NOT part of the author’s


explanation of the reason that utopian thinking is now
unfashionable?
D
) The failure of utopian social experiments in the nineteenth and early
twentieth centuries

The author makes no reference to the failed social utopias in the nineteenth and
early twentieth centuries. The reasons the author cites for why utopias are
unfashionable are lack of “a stable ground of generally accepted values to provide
the hard foundation on which to construct viable social systems, . . . the acceptance
by many scientists and sociologists of the view that the world of science and
technology sets its own ‘arising ends’ . . . [and a] tired resignation to the imperatives
of economics and scientific technology along with the collapse of the old
metaphysics”.

134. Implicit in the author’s ideas about the future and the role of science is the
belief that judgments about what is good or bad are:
D
) an appropriate part of scientific planning.

Concerning the need “to construct multiple models of possible futures”, the author
discusses “causative anticipations,” that is, how models of the future become agents
of change. This discussion implies the need for science to make value judgments
rather than to simply base the future on what is possible, and it further implies that it
is appropriate for science to do so: “Causative anticipations differ from predictions in
that the future they describe must not only be ‘possible’ but also embody
considerations of the ‘desirable.’ They imply value judgments as to what is desirable
or not, good or bad, and thus inevitably give a direction to the social and scientific
enterprise”.
135. The author expresses hope that society will impose pressure on the
scientific community to create a better future as a result of:
D
) greater focus on science as an agent of change.

The author argues that by means of “causative anticipations,” the scientific


community will become an agent of change. The author comments further: “The
social ferment that is beginning to agitate the community of scientists gives hope
that humanity still has a chance to control its destiny by imposing a direction on the
scientific endeavor and, in particular, by consciously planning the scientific
technology that will shape the modern world”.

136. As used in the passage, teleological most nearly means:


B
) purposive.

The author uses this word in the context of discussing how what is called
“progress” lacks purpose, which the author views as problematic: “We have
trained our social reflexes for technological ‘advances,’ however trivial their goals
and deleterious their long-range effects. Instead of conveying a teleological
quality, the word progress now means just moving on, even though the forward
motion is on a road that leads to disaster or despair”.

137. According to the passage, a precondition for effective scientific planning is


the:

A
) formulation of desirable social goals.

The author asserts the need for science to make value judgments in anticipating
the future so that science can be an effective agent for change because these
anticipations will have an actual impact: “Causative anticipations differ from
predictions in that the future they describe must not only be ‘possible’ but also
embody considerations of the ‘desirable.’ They imply value judgments as to what
is desirable or not, good or bad, and thus inevitably give a direction to the social
and scientific enterprise”.

Biological Sciences

Solution

Guess
140. What type or class of chemical messenger traveling in the blood would most
probably link the brain with the digestive tract and fat cells in the control of body
weight?

A
) Neurotransmitters

B
) Digestive enzymes

C
) Protein receptors
D
) Hormones

The question asks the examinee to identify the class of substances that would travel
through the bloodstream, linking the brain with the digestive tract. D is the best
answer because most hormones travel from the endocrine gland in which they were
produced, through the blood, to their target tissue. Neurotransmitters by definition
are released at neural synapses, although there are chemicals that act both at
synapses and like hormones in the bloodstream. Therefore, A is not the best answer.
Digestive enzymes are found in the digestive tract, but they do not travel through the
bloodstream. Therefore, B is incorrect. Protein receptors interact with chemical
messengers but are not the signal themselves. Therefore, C is not the best answer.
Thus, D is the best answer.

Solution

Guess
141. Which gene would produce a product that acts predominately on or in the cell
in which it is synthesized, ob or db?

A
) db because it encodes a hormone specific to fat cells

B
) db because it encodes a hormone receptor

The question asks the examinee to identify which gene would produce a product that
acts predominantly on or in the cell in which it is synthesized. The gene ob codes for
leptin, a peptide hormone. The gene db codes for the leptin receptor, which is located
on the surface of the cell. A is incorrect because it states that the gene product of db
is a hormone, but the product is actually the receptor. B is correct because it states
that the gene product of db is a hormone receptor, and hormone receptors act
predominantly on the cell in which they are produced. C is not the best answer
because hormones are secreted from the cells in which they are produced and most
hormones act on distal tissues. D is incorrect because ob encodes a hormone that
would be secreted from the fat cells. Thus, B is the best answer.

C
) ob because it encodes a hormone

D
) ob because it encodes a protein specific to fat cells

Solution

Guess
142. Which statement(s) in the passage link(s) leptin to weight regulation?

1. 40% of BMI is determined by genes.


2. The db gene in mice influences weight.
3. The ob gene in mice influences weight.

A
) I only

B
) II only

C
) I and III only

D
) II and III only

The question asks the examinee to identify which statements from the passage link
leptin to weight regulation, and three options are presented. Option I states that 40%
of BMI is determined by genes. Although leptin is a gene product, statement I does
not specifically link leptin to weight regulation. Option II states that the gene db
influences weight in mice. Because the gene db encodes leptin receptors, and
hormones exert their effect through the use of receptors, statement II links leptin
specifically to weight regulation. Option III states that the ob gene in mice influences
weight. Because the ob gene codes for leptin, this statement links leptin to weight
regulation. Of the answers provided, only D lists both options II and III. Thus, D is the
best answer.

Solution

Guess
143. What do both the set point hypothesis and the settling point hypothesis seek
to explain?

A
) How multiple, interacting factors determine body weight

The question asks the examinee to identify the characteristic that both the set point
and the settling point hypotheses try to explain. A is correct because both
hypotheses describe multiple factors that regulate body weight. B is incorrect
because both hypotheses list multiple interacting factors that influence body weight,
not individual factors acting independently. C is incorrect because only the settling
point hypothesis seeks to explain the effects of metabolism and environment
together. D is not the best answer because both hypotheses discuss the importance
of factors in addition to environment and behavior on body weight. Thus, A is the
best answer.
B
) How individual factors acting alone influence body weight

C
) How metabolism and the environment influence body weight

D
) How the environment and behavior influence body weight

Solution

Guess
144. Which hypothesis implies that a person can deliberately alter his or her own
body maintenance weight?

A
) The set point hypothesis because a thermostat can be reset

B
) The set point hypothesis because the set point can change with age

C
) The settling point hypothesis because, with the correct genotype, one’s
metabolism may allow weight to stabilize at a new level

The question asks the examinee to identify which hypothesis implies that a person
can deliberately alter his or her own body maintenance weight. A and B are incorrect
because the set point hypothesis suggests that the set point cannot be reset through
behavior modification. C correctly states that the settling point hypothesis allows the
set point to be reset through behavior modification. D is incorrect because it states
that diet and exercise cannot reset the set point, which contradicts the premise of the
settling point hypothesis. Thus, C is the best answer.

D
) The settling point hypothesis because diet and exercise cannot reset
the set point

Solution

Guess
145. One type of metabolic feedback loop that influences weight control involves
the regulation of glucose levels in the blood. Which organ in the digestive system
participates in this regulation by breaking down glycogen?

A
) Stomach

B
) Liver
The question asks the examinee to identify which organ in the digestive system
participates in regulation of glucose levels by breaking down glycogen. In order to
regulate blood glucose levels, glycogen is made, stored, and broken down in the liver
(B), not the stomach (A), the pancreas (C), or the small intestine (D). Thus, B is the
best answer.

C
) Pancreas

D
) Small intestine

Solution

Guess
146. In one study, diets of obese and lean subjects were adjusted so that members
of both populations either gained or lost weight. Subjects who lost weight used 15%
fewer calories than would be predicted to maintain their new weights. Subjects who
gained weight had to eat 15% more calories than would be predicted to maintain
their new weights. These results best support which hypothesis?

A
) The set point hypothesis because the metabolic changes seen would
favor a return to the subject’s original weight

The question presents the results of a study and asks the examinee to determine
which hypothesis is supported by those results. The results indicate that to maintain
their new weights, the people who had lost weight had to eat 15% fewer calories
than predicted and the people who had gained weight had to eat 15% more calories
than predicted. These data imply that a person’s weight is genetically determined;
the 15% discrepancy in the caloric consumption needed to maintain a new weight
suggests that metabolic changes favor a return to the subject’s original weight. This
best supports the set point hypothesis as described in A. B is incorrect because it
states that metabolic changes would not favor a return to the subject’s original
weight, the opposite of A. Answers C and D are incorrect, because they argue for the
settling point hypothesis, which is not best supported by the results. Thus, A is the
best answer.

B
) The set point hypothesis because the metabolic changes seen would
not favor a return to the subject’s original weight

C
) The settling point hypothesis because the experimental diet has a
positive effect on subjects with a genetic predisposition to obesity

D
) The settling point hypothesis because the experimental diet has a
positive effect on subjects without a genetic predisposition to obesity
Solution

Guess
147. A researcher studies sections of embryonic agnathan brains and discovers a
transitory stage in which a pituitary portal system develops. In regard to the
evolution of the vertebrate neuroendocrine axis, this finding best supports the
hypothesis that adult agnathans:

A
) represent a degenerative state.

The question describes the results of a study of embryonic agnathan brains and asks
the examinee to determine which hypothesis about adult agnathans is supported by
the results. According to the study, embryonic agnathan brains were found to have a
transitory stage in which a pituitary portal system develops even though, as
indicated in the passage, the adult agnathan brain does not have this system. A
correctly states that the experimental result supports the hypothesis that the adult
agnathans represent a degenerative stage. The embryological evidence suggests
that their ancestors had the portal system but that the structure has disappeared in
agnathans over evolutionary time. B is not the best answer because the embryonic
evidence suggests that the agnathan ancestral state had the portal system that
existing adult agnathans do not have. C is incorrect because adult agnathans, which
lack the portal system, do not resemble adult mammals, which have the portal
system. D is incorrect because adult agnathans, which lack the portal system, would
not resemble embryonic mammals that presumably must show evidence of the portal
system during embryogenesis in order to possess the portal system as adults. Thus,
A is the best answer.

B
) represent an ancestral state.

C
) resemble adult mammals.

D
) resemble embryonic mammals.

Solution

Guess
148. Suppose a newly discovered fossil agnathan skull (with its enclosed fossilized
brain) shows a fully vascular connection between the hypothalamus and the pituitary.
This finding supports which of the following statements about the evolution of the
vertebrate hypothalamic-pituitary axis?

A
) Ancient agnathans possess degenerate axes.

B
) Ancient agnathan axes are similar to existing bird axes.

The question proposes that a newly discovered fossilized agnathan skull shows a fully
vascular connection between the hypothalamus and pituitary, and then asks the
examinee to indicate which statement is supported by this finding. According to the
passage, all vertebrates except the existing agnathans have this vascular connection.
A is incorrect because ancient agnathans possess the portal system as do higher
vertebrates and therefore do not possess degenerate axes. B is correct because this
evidence supports the idea that ancient agnathans are similar to existing birds
(vertebrates) in having the vascular connection. C is incorrect because existing
agnathans, which lack a functional hypothalamic-pituitary system, are not similar to
the ancient agnathans (vertebrates), which are proposed to have the system. D is
incorrect because mammals, by having the vascular connection, are more similar to
ancient agnathans than to existing agnathans, which lack the vascular connection.
Thus, B is the best answer.

C
) Existing agnathan axes are similar to ancient vertebrate axes.

D
) Mammals are more similar to existing agnathans than to ancient
agnathans.

Solution

Guess
149. Which of the following is a negative feedback system involving the anterior
pituitary?

A
) LH suppression of estrogen release

B
) LH stimulation of GnRH release

C
) GnRH suppression of estrogen release

D
) Estrogen suppression of LH release

The question asks the examinee to identify which described process is a negative
feedback system involving the anterior pituitary. In this case, a negative feedback
system would be a regulatory loop in which a factor acts to suppress its own
synthesis or activity. A is incorrect because LH does not suppress the release of
estrogen, and if it did, this would not be a feedback mechanism. B is incorrect
because LH does not stimulate GnRH release, and if it did, this would be an example
of positive feedback. C is incorrect because GnRH does not suppress estrogen
release; GnRH stimulates estrogen release through a series of steps. D is correct for
the following reason: LH stimulates the release of estrogen from the ovaries. In turn,
the estrogen acts on the anterior pituitary gland to down-regulate the release of LH,
ultimately causing less estrogen to be released from the ovaries. This is an example
of a negative feedback loop. Thus, D is the best answer.

Solution

Guess
150. In the year 2010, a paleontologist provides convincing evidence that existing
agnathans have been incorrectly classified and should belong to an earlier and
previously undescribed vertebrate class. Fossils of this newly described class lack a
pituitary portal system. This finding suggests that the new class of vertebrates
(including agnathans):

A
) represents a degenerative state.

B
) represents an ancestral state.

The question asks the examinee to interpret new hypothetical evidence. The
evidence indicates that agnathans have been incorrectly classified and should belong
to a previously undescribed vertebrate class. Fossils of this new vertebrate class lack
a pituitary portal system, as do existing agnathans. This evidence does not suggest
that the new class represents a degenerative state because both the ancient and the
existing members of the class lack the pituitary portal system; therefore, A is
incorrect. The evidence does suggest that the new class represents an ancestral
state because both the existing and the ancient members of the class lack the
pituitary portal system; therefore, B is correct. C is incorrect because the
paleontologist gives evidence, including fossil evidence, that this class represents a
more ancient and not a newly derived state. D is incorrect because the new class
lacks the portal system, unlike other classes of vertebrates. Thus, B is the best
answer.

C
) represents a newly derived state.

D
) is similar to other classes of living vertebrates.

Solution

Guess
151. In the adult female rat, cyclical changes in sex steroids secreted by the
developing follicle switch to a positive feedback mechanism around the time of
ovulation. The positive feedback mechanism by which LH secretion is affected can
best be described as:

A
) an inhibition of LH by progesterone.

B
) an inhibition of LH by GnRH.

C
) a stimulation of LH by LH.

D
) a stimulation of LH by estrogen.

The question explains that cyclical changes in sex steroids secreted by the
developing follicle switch to a positive feedback system around the time of ovulation.
The question then asks the examinee to identify the positive feedback mechanism by
which LH secretion is affected. In this case, a positive feedback system would be a
regulatory loop in which LH acts to increase its own synthesis or activity. A is
incorrect because progesterone inhibition of LH secretion would not be a positive
feedback loop. B is incorrect because GnRH inhibition of LH secretion would not be a
positive feedback loop and GnRH is not a sex steroid. C is incorrect because LH
stimulation of LH secretion is not feedback involving a sex steroid. D is correct
because estrogen stimulation of LH secretion is a positive feedback loop involving a
sex steroid. Thus, D is the best answer.

Solution

Guess
152. In an adult female mammal, a greatly decreased production of FSH will most
likely result in a decrease in all of the following factors EXCEPT:

A
) progesterone production.

B
) estrogen production.

C
) GnRH production.

The question asks the examinee to determine which of the four effects will not occur
in response to a greatly decreased production of FSH. A and B are incorrect because
FSH stimulates progesterone and estrogen release; thus, a great decrease in FSH
should also result in a decrease in progesterone and estrogen. C is the best answer
because GnRH production is not stimulated by FSH; in fact, it is inhibited. Therefore,
GnRH production would not be expected to decrease as FSH decreases. D is incorrect
because Follicle Stimulating Hormone stimulates follicle maturation; therefore, a
great decrease in FSH should result in a decrease in follicle maturation as well. Thus,
C is the best answer.

D
) follicle maturation.
Solution

Guess
153. Which of the following is most likely the function of the ice-water bath in the
passage?

A
) To prevent the reaction mixture from being heated above 90°C

B
) To prevent impurities from evaporating from the reaction flask

C
) To aid in the condensation of the distillate

Cooling the distillate reduces the vapor pressure and facilitates the condensation of
the distillate. Thus, C is the best answer.

D
) To aid in the evaporation of the distillate

Solution

Guess
154. What is the maximum number of moles of cyclohexene that can be formed by
the reaction in the passage?

A
) 0.01

B
) 0.1

The maximum number of moles of cyclohexene that can be produced is equal to the
number of moles of cyclohexanol used. The number of moles of cyclohexanol used
can be calculated by dividing the mass of cyclohexanol used, which was 10 g, by the
molar mass of cyclohexanol, 100 g/mol. Thus, B is the best answer.

C
) 1.0

D
) 10.0

Solution
Guess
155. Which of the following was the most likely reason for washing the distillate
with aqueous Na2CO3?

A
) To neutralize any remaining acid

Because Na2CO3 is a base, it will react with any H3PO4 in the distillate. Thus, A is the
best answer.

B
) To react with any remaining cyclohexanol

C
) To protonate the carbocation intermediate

D
) To destroy the excess magnesium sulfate

Solution

Guess
156. The students were careful to keep the temperature of the reaction flask under
90°C to prevent:

A
) the unreacted cyclohexanol from distilling.

The students did not want unreacted cyclohexanol to be distilled with the product.
Thus, A is the best answer.

B
) the product cyclohexene from distilling.

C
) the product cyclohexene from decomposing.

D
) rearrangement of the carbocation intermediate.

Solution

Guess
157. The difference between the boiling points of cyclohexanol and cyclohexene
most likely can be attributed to which of the following factors?

A
) Steric hindrance

B
) Orbital hybridization

C
) Hydrogen bonding

Cyclohexanol has stronger intermolecular forces because it is capable of hydrogen


bonding through the –OH group and therefore has a higher boiling point. Thus, C is
the best answer.

D
) Thermal stability

Solution

Guess
158. The students started with 0.25 mol of cyclohexanol and isolated 21.3 mL of
cyclohexene. Which of the following expressions shows the number of moles of
cyclohexene isolated by the students?

A
) (21.3 × .811/82)(100/.25)

B
) 21.3/.811

C
) 21.3 × .811 × 82

D
) (21.3 × .811)/82

The number of moles of cyclohexene isolated can be calculated by determining the


mass in grams of cyclohexene isolated (volume × density), then converting to moles
by dividing by the molar mass:

(21.3 mL × 0.811 g/mL)/(82 g/mol)

Thus, D is the best answer.

Solution

Guess
159. Which of the following organelles is(are) found in both bacterial and
eukaryotic cells?

1. Plasma membrane
2. Mitochondrion
3. Endoplasmic reticulum

A
) I only

The question asks the examinee to identify which organelles are found in both
bacterial and eukaryotic cells. Of the three options listed, only the plasma membrane
(Option I) is found in both types of cells; thus, A is correct because it lists Option I
only. The other two options include mitochondrion and endoplasmic reticulum, both
of which are found in eukaryotic cells, but not in bacterial cells. B, C, and D are
incorrect because they include either the mitochondrion or the endoplasmic
reticulum. Thus, A is the best answer.

B
) II only

C
) I and II only

D
) I and III only

Solution

Guess
160. Exponential growth of the bacterial population in Figure 1 occurs between
points:

1. A and B.
2. B and C.
3. C and D.

A
) I only

B
) III only

C
) I and II only

D
) I and III only

The question asks the examinee to use a graph in the passage to identify the time
points between which a bacterial population underwent exponential growth. Option I
is the time period between A and B, the first phase in which an exponential increase
occurred in the number of viable bacteria. Option II is the time period between B and
C, an intermediate phase in which a considerable decrease occurred in viable cell
number after the administration of an antibiotic. Option III is the time period between
C and D, the second phase in which an exponential increase occurred in the number
of viable bacteria. Only response D properly lists both time periods during which
exponential growth occurred, as indicated by options I and III. Thus, D is the best
answer.

Solution

Guess
161. Which of the following compounds could contribute to the nitrogen
requirement in the growth medium?

A
) Glucose

B
) Glycerol

C
) Glycine

The question asks the examinee to identify which compound in the growth medium
could contribute to the nitrogen requirement of the bacteria. Three of the four
substances listed, glucose (A), glycerol (B), and n-hexanoic acid (D), contain only
carbon, oxygen, and hydrogen, and therefore cannot contribute nitrogen. Only the
amino acid glycine (C) contains nitrogen. Thus, C is the best answer.

D
) n-Hexanoic acid

Solution

Guess
162. If antibiotic Z acts by inhibiting translation, its cellular site of action would
most likely be the:

A
) nucleus.

B
) cell membrane.

C
) lysosomes.

D
) ribosomes.
The question asks the examinee to determine the site of antibiotic Z action, given
that this antibiotic inhibits translation. A, B, and C are incorrect because translation
does not occur in the nucleus, on the cell membrane, or in the lysosomes. Translation
occurs on the ribosomes. Thus, D is the best answer.

Solution

Guess
163. The approximate generation time (doubling time) observed between points A
and B in Figure 1 is:

A
) 30 min.

The question asks the examinee to identify the approximate generation time
(doubling time) for the bacteria between points A and B on the graph in the passage.
In this graph the total cell number can be most precisely correlated with time at 6
and 7 h. The cell number was 100,000 at 6 h and 400,000 at 7 h. There were two
doublings, 100,000 to 200,000 and 200,000 to 400,000, in the one hour between
hours 6 and 7. Therefore, the doubling time for this culture was 30 min, the answer
given in A. Thus, A is the best answer.

B
) 1 h.

C
) 2 h.

D
) 5 h.

Solution

Guess
164. A semilogarithmic plot of the data used to generate Figure 1 would look like
which of the following graphs?

A
)

B
)

C
)
D
)

The question asks the examinee to identify a semilogarithmic plot of the data used to
generate Figure 1. A, B, and C are incorrect because the data are still plotted using a
linear scale on both axes; in A, the plot of cell number relative to time is reversed; in
B, the data are plotted using a bar graph instead of a line graph; and in C, negative
values have been added to the y axis. D is correct because it shows a
semilogarithmic plot with the cell number plotted on a logarithmic scale and the time
plotted in a linear scale. Thus, D is the best answer.

Solution

Guess
166.
The above graph represents an action potential recorded from the cell body of a
neuron. What type of ion movement is causing the depolarization of the neuronal
membrane at the time denoted by the arrow?

A
) Sodium ions are moving into the neuron down a concentration
gradient.

The question asks the examinee to identify the type of ion movement that occurs
during the rising portion of the action potential shown in the figure. At the point
indicated on the graph, sodium ions are moving into the neuron, causing the
membrane potential to become more positive. A is correct in stating that this effect is
being caused by sodium ions that are moving into the neuron down the concentration
gradient at this point. B is incorrect because this effect is not being caused by sodium
ions that are moving out of the neuron via active transport. C and D are incorrect
because the change in potential at the point indicated on the graph is not caused by
movement of potassium ions. Thus, A is the best answer.

B
) Sodium ions are being moved out of the neuron via active transport.

C
) Potassium ions are moving out of the neuron down a concentration
gradient.

D
) Potassium ions are being moved into the neuron via active transport.

Solution

Guess
167.

Compound A is shown below.


Compound A

Which of the following structures has a configuration that is identical to that of


Compound A?

A
)

B
)

C
)

D
)

The relative positions of the four substituents around the chiral carbon atom are the
same in the figure in D as in Compound A. Also, the –OH and –COOH groups extend
away from the viewer, and the hydrogen and –CH2OH group extend toward the
viewer both in Compound A and in the figure in D. Thus, D is the best answer.

Solution

Guess
168. Albumin is the major blood osmoregulatory protein. The most likely effect of a
sharp rise in the level of serum albumin is:

A
) a drop in blood pressure.

B
) an increase in immune response.

C
) an efflux of albumin into the interstitial fluids.

D
) an influx of interstitial fluid into the bloodstream.

The question asks the examinee to identify the most likely effect of a sharp rise in the
level of serum albumen, a major blood osmoregulatory protein. A sharp rise in
osmotically active albumin in the serum would increase the flow of interstitial fluid
into the bloodstream and result in an increase in blood pressure, not a decrease.
Therefore, A is incorrect, and D is correct. B is incorrect because the immune
response does not depend on the level of serum albumen. C is incorrect because
albumen would not normally pass through capillary walls. Thus, D is the best answer.

Solution

Guess
169. The osmotic concentration of plasma proteins in the venous side of capillaries
helps reduce the amount of interstitial fluid in tissues by inducing:

A
) passive H2O diffusion along a concentration gradient.

The question asks the examinee to identify why the osmotic concentrations of
plasma proteins on the venous side of capillaries helps reduce the amount of
interstitial fluid. A is correct because it is the passive movement of water into the
bloodstream that will most directly reduce the amount of interstitial fluid. B and C are
incorrect because the effect of the plasma proteins is not to induce either passive or
facilitated ion transport. D is incorrect because the water would not move by active
transport. Thus, A is the best answer.

B
) passive ion diffusion along an electrochemical gradient.

C
) facilitated ion transport along an electrochemical gradient.

D
) active H2O transport mediated by an ATP- dependent pump.

Solution

Guess
170. Capillaries in the kidney and elsewhere in the body maintain fluid homeostasis
by balancing hydrostatic and osmotic pressures. Which of the following is the initial
effect of a blood clot forming on the venous side of a capillary bed?

A
) Net fluid flow in the direction of interstitial spaces will increase.

After introducing the idea of fluid homeostasis, which is obtained by balancing


hydrostatic and osmotic pressures, the question asks the examinee to predict the
consequences of a blood clot on the venous side of a capillary bed. To answer this, it
is necessary to know that blood flows from arteries to capillaries and then to veins. If
flow is blocked at the venous side, blood would accumulate in the capillaries. Thus,
hydrostatic pressure would build up in the capillaries, causing a net increase in fluid
flow into the interstitial spaces. A is therefore correct. B is incorrect because there will
be an increase in net fluid flow into the interstitial spaces, not a decrease. C is not
the best answer because the increase in osmotic pressure in the capillaries would be
an indirect result of the fluid flow out of the capillaries. D is incorrect because the
flow of fluid out of the capillaries would slightly increase, not decrease, the osmotic
pressure. Thus, A is the best answer.

B
) Net fluid flow in the direction of interstitial spaces will decrease.

C
) Capillary osmotic pressure will increase.

D
) Capillary osmotic pressure will decrease.

Solution

Guess
171. Which of the following changes in flow rate or in solute concentrations would
NOT occur if the blood inflow rate were increased, increasing the pressure in the
dialysis chamber?

A
) The blood volume reaching the outflow tube per unit time would
increase.

B
) The osmotic concentration of proteins in the dialysate fluid would
increase.

The question asks the examinee to identify which changes in flow rate or solute
concentrations would NOT occur if the blood inflow rate were increased, consequently
increasing the pressure in the dialysis chamber. The illustrations in the passage help
clarify the location of the blood inflow, which comes from the patient and enters the
dialysis machine. A, C, and D are incorrect because they identify changes that would
occur. Blood volume reaching the outflow tube would increase per unit time (A).
Osmotic concentrations of proteins in the blood would increase or remain the same
because proteins are generally too large to pass through the membrane (C). The
filtration rate across the membrane would increase because the pressure in the
chamber would be greater (D). However, the osmotic concentration of proteins would
not be expected to increase in the dialysate fluid because proteins generally do not
pass through the membrane. Thus, B is the best answer.

C
) The osmotic concentration of proteins in the blood outflow would
increase or remain unchanged.

D
) The filtration rate across the dialysis membrane would increase.
Solution

Guess
173. Why are high concentrations of sodium included in the dialysate (Table 1)?

A
) To induce water movement from the blood into the dialysate fluid

B
) To maintain a high osmotic pressure in the dialysate solution

C
) To maintain isotonicity of the dialysate solution with blood

The question asks the examinee to correctly identify the reason why high
concentrations of sodium are included in the dialysate. As shown in Table 1, the
dialysate contains 128–140 mEq/L of sodium, which is equivalent to the sodium
concentration in normal plasma, as well as to the sodium concentration in the blood
of a patient with renal failure. Consequently, A is incorrect because solvent will not
move in response to the sodium concentration in the dialysate, if it is the same as the
concentration in the patient’s blood. B is incorrect because the sodium in the
dialysate is not maintaining a high osmotic pressure if the patient’s blood and the
dialysate have the same sodium concentration. C is correct because examination of
the concentrations of all ions in plasma and dialysate suggests that sodium levels
must remain high in the dialysate so that the dialysate is isotonic to blood. D is
incorrect because the sodium concentration of the dialysate would not compensate
for the levels of urea nitrogen and creatinine nitrogen in the blood. Thus, C is the best
answer.

D
) To compensate for the urea nitrogen and creatinine in the blood

Solution

Guess
174. Which of the following figures (A–D) shows expected solute filtration rates
(mEq/mL-min) as a function of molecular weight for two dialysis membranes:
Membrane 1 with large pores and Membrane 2 with small pores?

A
)

B
)

C
)

D
)
The question asks the examinee to select the graph that best illustrates how solute
filtration rates vary as a function of solute molecular weight for dialysis membranes
with large pores (Membrane 1) and small pores (Membrane 2). As molecular weight
increases, the rate of filtration is expected to decrease, as shown in C and D. A and B
are incorrect because they show the rate of filtration increasing as the molecular
weight increases. Because Membrane 1 has larger pores than Membrane 2,
Membrane 1 is expected to allow more molecules to pass at any given molecular
weight. D is correct because it shows the trend of decreasing filtration rate with
increasing molecular weight, and it shows that Membrane 1 (with larger pores) has a
higher filtration rate than Membrane 2 for any molecular weight. Thus, D is the best
answer.

Solution

Guess
175. A person with genotype IAIASs will express the A antigen in which of the
following places?

A
) In saliva and on erythrocytes

The question asks the examinee to identify the location in which the A antigen will be
expressed in a person with genotype IAIASs. The I locus determines the identity of
the antigen displayed on the erythrocyte surface, in this case A. The S locus
determines if the same antigen (A) will be secreted into saliva and, if the individual is
male, into semen. Because the individual contains the dominant S allele, the A
antigen will be secreted into these fluids. A correctly identifies that the A antigen will
be found both in the saliva and on the erythrocytes of a person with genotype IAIASs.
B is incorrect because it excludes both locations of expression. C is incorrect because
it lists erythrocytes only. D is incorrect because it lists saliva only. Thus, A is the best
answer.

B
) Neither in saliva nor on erythrocytes

C
) Only on erythrocytes

D
) Only in saliva

Solution

Guess
176. Ulex extract most likely reacts with what component of type O blood?

A
) Antigen A

B
) Substance H

The question asks the examinee to identify the component of type O blood that
reacts with Ulex extract. Although Ulex extract reacts with type O blood, it does not
react with Bombay blood type. A person with Bombay blood type does not produce
any substance H. A person with type O blood does produce substance H but lacks the
enzymes needed to convert substance H into either the A or B antigens. Because
people with type O blood have substance H and people with Bombay blood type do
not, the extract most likely interacts with substance H, implying that B is correct. A is
incorrect because a person with blood type O would not have A antigens. C is
incorrect because secretor gene products are independent of ABO blood type. D is
incorrect because the i allele produces a functionless enzyme that is unable to
catalyze the conversion of substance H and would therefore not produce any
products. Thus, B is the best answer.

C
) Secretor gene antigens

D
) Enzyme products of the i allele

Solution

Guess
177. A person with the genotype hhSS would express which of the following
antigens?

A
) A and B in saliva, but not on erythrocytes

B
) A and B on erythrocytes, but not in saliva

C
) Neither A nor B antigens

The question asks the examinee to identify which antigens would be produced by
someone with the genotype hhSS. A person with the genotype hh cannot produce
substance H. Regardless of the genotype at the I locus, this person will not produce
either antigen A or antigen B because both antigens are produced from substance H.
Because this person does not produce antigens A and B, there are no blood antigens
to secrete into the saliva and semen, so the genotype at the S locus is irrelevant. C
correctly states that neither A nor B antigens will be produced. A, B, and D are
incorrect because they state that the person is able to produce A or B antigens or
both. Thus, C is the best answer.

D
) B on erythrocytes only
Solution

Guess
178. Which of the following best explains why adults with the Bombay blood type
lack the A and B antigens?

A
) Such individuals lack the I&nbspA and I&nbspB alleles.

B
) Such individuals lack substance H.

The question asks the examinee to identify the best explanation for why adults with
Bombay blood type lack the A and B antigens. People with Bombay blood type
(genotype hh) cannot produce substance H. Because antigens A and B are modified
versions of substance H, a person with Bombay blood type also cannot produce
antigens A and B, supporting choice B. A is incorrect because a person with Bombay
blood type can have the IA and IB alleles; it is the lack of substance H that prevents
expression of those phenotypes. C and D are incorrect because the passage states
that the h allele is inactive, implying that either the allele is not expressed or that its
product is nonfunctional and therefore could not have an effect on the I locus or its
products. Thus, B is the best answer.

C
) Expression of the h allele inactivates the enzymes produced by
I&nbspA and I&nbspB.

D
) Expression of the h allele suppresses expression of the I gene.

Solution

Guess
179. If Ulex extract is added to blood from a type A donor, will clumping of
erythrocytes occur?

A
) Yes, but only if the donor’s genotype is I&nbspA i

The question asks the examinee to determine whether clumping will occur if the Ulex
extract is added to blood from a type A donor. According to the passage, clumping
occurs when the extract is added to type O blood, but not when it is added to blood
with the Bombay phenotype. The difference between type O blood and Bombay blood
type is the ability to produce substance H, indicating that the Ulex extract most likely
reacts to substance H. A person with genotype IAi still has a significant amount of
substance H present. However, a person with genotype IAIA converts almost all
substance H into A antigen. Therefore, clumping would most likely occur in people
with genotype IAi but not IAIA, which makes A the most reasonable answer. B is not
the best answer because a person with IAIA has little to no substance H. C and D are
incorrect because clumping will occur in at least some type A individuals. Thus, A is
the best answer.

B
) Yes, but only if the donor’s genotype is I&nbspAI&nbspA

C
) No, because the erythrocytes lack the A antigen

D
) No, because the

Solution

Guess
180. Compound II can also be prepared by treatment of 2,3-dimethyl-2-butanol
with:

A
) Br2/light.

B
) Br2/CCl4.

C
) NaBr.

D
) HBr.

Alcohols can be converted to alkyl bromides by the reaction with hydrogen bromide.
Treatment of 2,3-dimethyl-2-butanol with hydrogen bromide would result in the
formation of 2-bromo-2,3-dimethylbutane. Thus, D is the best answer.

Solution

Guess
181. Which of these cycloalkanes will undergo free radical bromination most
rapidly?

A
)

B
)

A tertiary free radical is more stable than a secondary free radical, and a secondary
free radical is more stable than a primary free radical. Because free radical
bromination requires the formation of an alkyl radical, the reaction that proceeds
through the most stable radical will take place most rapidly. Methylcyclohexane
would produce a tertiary free radical. Thus, B is the best answer.

C
)

D
)

Solution

Guess
182. According to the gas chromatographic data in Table 1, what is the composition
of the reaction mixture from Reaction 1?

A
) 33% II and 67% III

Peak 1 with a retention time of 60 s is due to Compound II, and Peak 2 with a
retention time of 74 s is due to Compound III. Peak 1 has half the peak area of Peak 2,
indicating that the compounds II and III are present in the reaction mixture in a 1:2
ratio. Thus, A is the best answer.

B
) 33% III and 67% II

C
) 45% II and 55% III

D
) 45% III and 55% II

Solution

Guess
183. What stereochemical outcome is expected in the formation of Compound II?

A
) It would be formed as a pair of enantiomers, because invasion of the
bromine radical can approach from either side.

B
) It would be formed as a pair of enantiomers, because inversion is
faster than carbon–bromine bond formation.

C
) It would be formed as a pair of diastereomers, because the bromine
radical can approach from either side.

D
) It is an optically inactive compound, because it has no chiral carbon.

Compound II contains no chiral carbon atoms and is therefore not optically active.
Thus, D is the best answer.

Solution

Guess
184. The 1H NMR of Compound II would consist of:

A
) two singlets.

B
) a doublet and a septet.

C
) a singlet, a doublet, and a septet.

The 1H NMR spectrum of Compound II would contain a singlet due to the methyl
groups attached to C-2, a doublet due to the methyl groups attached to C-3, and a
septet due to the hydrogen attached to C-3. Thus, C is the best answer.

D
) two singlets, a doublet, and a septet.
Solution

Guess
187. Which of the following structures is the enol form of pyruvate?

A
)

B
)

The enol form of pyruvate forms when one of the methyl hydrogen atoms is
transferred to the ketone oxygen atom, resulting in a double bond between C-2 and
C-3. Thus, B is the best answer.

C
)

D
)

Solution

Guess
188.

A reaction in the citric acid cycle is shown below. (The equation is not balanced.)

This reaction would be described as:

1. oxidation–reduction.
2. decarboxylation.
3. isomerization.

A
) I only

In this reaction, the alcohol functionality is converted to a ketone functionality by an


oxidizing agent. It is therefore an oxidation–reduction reaction. Thus, A is the best
answer.

B
) II only

C
) Both I and II

D
) Both I and III

Solution

Guess
189.

The four five-carbon carbohydrates shown below illustrate principles of carbohydrate


nomenclature.
Another five-carbon carbohydrate is xylulose. Which of the following statements
apply(applies) to xylulose?

1. It is an isomer of deoxyribose.
2. It is an isomer of ribose.
3. It is an isomer of xylose.

A
) I only

B
) II only

C
) I and III only

D
) II and III only

The figures in this item show that ribose, ribulose, and xylose are all isomers with the
same molecular formula, but that deoxyribose has a different molecular formula. It is
reasonable therefore to conclude that xylulose would be an isomer of xylose and
ribose. Thus, D is the best answer.

Solution

Guess
190. One characteristic common to arteries, veins, and capillaries is the:

A
) presence of a layer of endothelial cells.

The question asks the examinee to identify one characteristic common to arteries,
capillaries, and veins. A is correct because all three types of vessels possess an inner
layer of endothelial cells. B is incorrect because only veins have valves. C is incorrect
because only certain types of arteries dilate or constrict to regulate blood flow. D is
incorrect because the exchange of nutrients with the surrounding tissues occurs only
in capillaries. Thus, A is the best answer.

B
) presence of numerous valves that prevent the backflow of blood.

C
) ability to actively dilate or constrict in regulating blood flow.

D
) ability to supply surrounding tissues with nutrients by filtration and
diffusion.
Solution

Guess
191. Tissue that is very active metabolically, such as skeletal muscle, contains
large numbers of:

A
) nuclei.

B
) fat deposits.

C
) blood capillaries.

The question asks the examinee to identify the feature that is abundant in
metabolically active tissue. Metabolically active tissue requires a large supply of
oxygen and produces many waste products. Such tissue needs ample capillary blood
flow to provide oxygen and to remove waste products. As a result, the best answer is
C. A (nuclei) is incorrect because although mature skeletal muscle cells have multiple
nuclei, these nuclei result from the fusion of individual muscle cells during muscle
development, each of which contributes one nucleus. Thus, the total number of
nuclei in the muscle itself is no greater than if a muscle were composed of individual
cells. B is incorrect because even though fat deposits are useful for long-term energy
storage, metabolically active skeletal muscle relies primarily on glycogen stores that
provide energy in a more easily utilized form. Answer D is incorrect because
lymphatic vessels do not deliver materials needed for metabolism. Thus, C is the best
answer.

D
) lymphatic vessels.

Solution

Guess
192. The finches observed by Darwin on the Galapagos Islands are an example of
adaptive radiation. In order to set up conditions that would produce adaptive
radiation, it would be necessary to place members of:

A
) one species in one rapidly changing environment.

B
) one species in several different environments.

The question refers to the finches of the Galapagos Islands as an example of adaptive
radiation and asks the examinee to identify the conditions required for adaptive
radiation. Adaptive radiation involves the divergence of one species into multiple
species over time, which can occur when subgroups of the original species are
separated or isolated in different environments so that these subgroups evolve
independently of one another. B is correct because it describes this situation. A is not
the best answer because a single rapidly changing environment does not encourage
adaptive radiation. Rather, as the environment changes, the species would be more
likely to collectively evolve into a single new species adapted to the new
environment. C and D are incorrect because adaptive radiation begins with one
species, not several, regardless of whether the species are similar (C) or unrelated
(D). Thus, B is the best answer.

C
) several very similar species in the same environment.

D
) several unlike species in one environment to compete for the same
resources.

Solution

Guess
193.

Which of the compounds shown below can form hydrogen bonds with water?
CH3–CH2–CH3
Compound 1 CH3–CH2–NH2
Compound 2

A
) Compound 1 only

B
) Compound 2 only

In Compound 2, which is an amine, two hydrogen atoms are covalently bonded to the
nitrogen atom, and nitrogen is an electronegative element. Compound 1, an alkane,
does not contain any hydrogen atoms covalently bonded to an atom of an
electronegative element. Thus, B is the best answer.

C
) Both compounds

D
) Neither compound

Solution

Guess
194. The term α-helix refers to what kind of protein structure?

A
) Primary

B
) Secondary

The term α-helix describes a helical conformation of a protein chain that is created by
hydrogen bonding between groups on the protein backbone. This is an example of
the secondary structure of a protein. Thus, B is the best answer.

C
) Tertiary

D
) Quaternary
Solution

Guess
195. Which conclusion about Peptide A receptor binding can be reached from
Figure 1?

A
) Peptide A requires 100% receptor binding to produce the most
osteoblatin.

B
) Peptide A produces the most osteoblatin possible at less than 100%
receptor binding.

The question asks the examinee to identify the conclusion that can be drawn about
Peptide A receptor binding based on the results presented in Figure 1. Figure 1 shows
the percentage of Peptide A binding to receptors (solid line) and the concentration of
osteoblatin produced (dashed line) as a function of Peptide A concentration. As the
concentration of Peptide A increases, so does the percentage of Peptide A binding
and the concentration of osteoblatin. However, the curve for osteoblatin
concentration levels off at a Peptide A concentration of about 25 ng/mL, while the
curve for the percentage of Peptide A receptor binding slows and begins to level off
at approximately twice this concentration. When Peptide A concentration is 25 ng/mL,
about 50% of Peptide A has bound to receptors. Because the maximal production of
osteoblatin occurs when only 50% of the Peptide A receptors are occupied, B is the
best answer. A is incorrect because maximal osteoblatin production occurs at 50%
receptor binding; 100% receptor binding is not required. C is incorrect because 50%
binding of Peptide A results in maximal osteoblatin production, not 50% production. D
is incorrect because there is no increased synthesis of osteoblatin at over 50%
receptor binding. Thus, B is the best answer.

C
) At 50% binding to its receptors, Peptide A produces 50% of the
maximal amount of osteoblatin.
D
) Peptide A continues to stimulate increasing levels of osteoblatin
production at over 50% receptor binding.

Solution

Guess
196. When OB is incubated with a constant concentration of Peptide A (25 ng/mL)
and increasing concentrations of Peptide B, osteoblatin concentration:

A
) increases exponentially.

B
) increases linearly.

C
) remains constant.

D
) decreases.

The question asks the examinee to identify the relationship between osteoblatin
concentration and increasing concentrations of Peptide B when osteoblasts are
incubated with a constant concentration of Peptide A. Peptide A stimulates the
production of osteoblatin, but Peptide B does not. Furthermore, Peptide B
competitively inhibits Peptide A by binding to the same receptor sites on the cell
membranes. As the concentration of Peptide B increases, more and more receptor
sites are occupied by Peptide B rather than by Peptide A. This prevents Peptide A
from being able to activate expression of osteoblatin, therefore decreasing the
osteoblatin concentration (D). A, B, and C are incorrect because the osteoblatin
concentration does not increase exponentially (A), increase linearly (B), or remain
constant (C). Thus, D is the best answer.

Solution

Guess
197. What can be concluded about Peptide A from Experiment 2?

A
) It is ineffective in stimulating osteoclastic activity.

B
) It acts directly on OC to increase bone resorption.

C
) It acts indirectly through OB to increase bone resorption.
The question asks the examinee to identify the conclusion that can be drawn about
Peptide A based on the results of Experiment 2. The first key observation from the
experiment is that Peptide A stimulates the synthesis of osteoblatin, but only when
OB are present. The second key observation is that OC activity cannot be stimulated
by the presence of either Peptide A or OB alone; only when both Peptide A and OB
are present is OC activity stimulated. This implies that Peptide A functions indirectly
to increase bone resorption by stimulating OB to produce osteoblatin, which in turn
stimulates OC activity (C). A is incorrect because Peptide A does stimulate OC activity
through its interaction with OB. B is incorrect because Peptide A must interact with
OB in order to increase OC activity. D is incorrect because although Peptide A acts
directly on OB, it stimulates the production of osteoblatin, not bone formation (Table
1, Group 2). Thus, C is the best answer.

D
) It acts directly on OB to increase bone formation.

Solution

Guess
198. In Experiment 2, which three groups demonstrated cell line viability and
allowed the determination of basal cellular activities?

A
) Groups 1, 2, and 3

B
) Groups 1, 3, and 5

The question asks the examinee to identify which three groups in Experiment 2
demonstrated cell line viability and allowed the determination of basal cellular
activities. Essentially, this question is asking the examinee to identify the three
control groups—the groups that provide the basis for comparison to see the effects of
Peptide A. The correct answer is B because the control groups include all groups in
which Peptide A was omitted. This includes groups 1, 3, and 5. A, C, and D are
incorrect because each contains at least one experimental group that includes
Peptide A. Thus, B is the best answer.

C
) Groups 2, 4, and 6

D
) Groups 4, 5, and 6

Solution

Guess
199. To most effectively study the effect of Peptide A on transcription of the
osteoblatin gene, a scientist should determine the:

A
) sequence of the osteoblatin gene.

B
) amount of osteoblatin mRNA in the cell.

The question asks the examinee to identify the most effective method that could be
used to study the effect of Peptide A on transcription of the osteoblatin gene. RNA is
the direct product of transcription; therefore, the amount of osteoblatin mRNA in a
cell would be a direct measure of transcription of the osteoblatin gene (B). A is
incorrect because regulatory sequences within the DNA only indirectly indicate the
amount of mRNA the cell will produce. C is not the best answer because the amount
of osteoblatin peptide in the cell would be a direct indicator of translation but only an
indirect indicator of transcription. D is incorrect because the total amount of RNA in
the cell would depend on the transcription of many genes, not just the gene that
codes for osteoblatin. Thus, B is the best answer.

C
) amount of osteoblatin peptide in the cell.

D
) total RNA in the cell.

Solution

Guess
200. The overall effect of Peptide A on bone is to:

A
) increase bone mass.

B
) decrease bone mass.

The question asks the examinee to identify the effect of Peptide A on bone. The
passage states that Peptide A binds to specific sites on the plasma membrane of
osteoblasts (OB), the bone-forming cells. Experiment 1 results show that the effect of
Peptide A on OB was to increase secretion of osteoblatin, which markedly stimulates
bone resorption. This indicates that the overall effect of Peptide A on bone is to
decrease bone mass (B). A is incorrect because bone mass decreases rather than
increases. C is incorrect because there is no evidence to support the idea that
haversian canals undergo change. D is incorrect because bone mass decreases rather
than stays the same. Thus, B is the best answer.

C
) narrow haversian canals.
D
) maintain constant bone mass.
Solution

Guess
201. According to the passage, which of the following compounds can serve as an
antioxidant?

A
)

B
)

Based on the passage, to serve as an antioxidant, the compound must have a


hydrogen atom that can be removed to form a radical and an aromatic ring that will
permit subsequent coupling with a peroxy radical. A hydrogen atom attached to the
nitrogen in the compound in B can be removed, and the resulting radical would be
able to couple with the peroxy radical at position 4. Thus, B is the best answer.

C
)

D
)

Solution

Guess
202. Compared to the activation energy for the hydrogen-atom abstraction from
BHT, the activation energy for the reaction between a BHT radical and peroxy radical
is:

A
) less.

The passage states that the rate-controlling step in the antioxidation process is the
formation of the BHT radical, indicating that the reaction of the BHT radical with the
peroxy radical occurs more quickly. Thus, A is the best answer.

B
) equal.

C
) greater.

D
) impossible to determine from the data given.
Solution

Guess
203. Which of the following phenols can undergo intra- molecular hydrogen
bonding?

A
)

B
)

C
)

D
)

The compound in D can form intramolecular hydrogen bonds between the adjacent –
OH and –CO2H groups. Thus, D is the best answer.

Solution

Guess
204. A 1H NMR spectrum of BHT contains how many unique (nonequivalent) proton
signals?

A
) 4

In BHT, the 18 protons in the tert-butyl groups at positions 2 and 6 are equivalent,
giving rise to one signal. The protons in the methyl group at position 4 give rise to a
second signal. The ring protons at positions 3 and 5 are equivalent, resulting in a
third signal, and the –OH proton gives rise to a fourth signal. A 1H NMR spectrum of
BHT will contain 4 unique signals. Thus, A is the best answer.

B
) 5

C
) 12

D
) 24
Solution
Guess
205. When used as described in the passage, antisense drugs prevent:

A
) DNA replication.

B
) RNA transcription.

C
) RNA translation.

The question asks the examinee to identify the effect of an antisense drug. When
antisense mRNA molecules are used as described in the passage, the antisense
molecules bind specifically to the sense mRNA, preventing the process of translation.
As a result, C is the best answer. A, B, and D are not the best answers because the
passage describes the effect of antisense RNA binding to mRNA, and mRNA is directly
involved in translation. However, because RNA can hybridize with either RNA or DNA,
a role for antisense RNA in the regulation of DNA replication, transcription, and even
cell replication may be possible, but these processes are not the focus of the
passage. Thus, C is the best answer.

D
) cell replication.

Solution

Guess
206. To be an effective therapy, an antisense gene that is incorporated into a
genome that contains the target gene must be:

A
) on the same chromosome as the target gene but not necessarily be
physically adjacent.

B
) on the same chromosome as the target gene and must be physically
adjacent.

C
) regulated in a similar manner as the target gene.

The question focuses on antisense genes that are incorporated into the genome of
their target gene and asks the examinee to identify which antisense gene
characteristic would be required for the gene to be therapeutically effective. To
provide effective therapy, this antisense gene would need to be regulated in a
manner similar to the manner in which the target gene is regulated so that the
antisense RNA is produced at the same time that the sense mRNA is produced. This
would ensure that the antisense RNA is available to bind with the sense mRNA,
thereby preventing its subsequent translation. As a result, C is correct. A, B, and D
are incorrect because the key feature determining whether an antisense drug will
work is the timing of the expression of the antisense gene. This is controlled by
specific regulatory elements, not necessarily the location of the antisense gene
relative to the target gene. Thus, C is the best answer.

D
) coded on the same strand of DNA as the target gene.

Solution

Guess
207. Phenylketonuria is a genetic disorder caused by a mutation in the gene for the
enzyme phenylalanine hydroxylase, which eliminates its enzymatic activity. Could an
antisense drug help individuals with this disorder?

A
) Yes, if it binds to the mRNA of the phenylalanine hydroxylase gene and
prevents its translation

B
) Yes, if it is incorporated into the chromosome and prevents the
expression of the phenylalanine hydroxylase gene

C
) No, because mRNA does not persist in the cytoplasm of the cell

D
) No, because blockage of phenylalanine hydroxylase gene expression
will not remedy the original disorder

The question indicates that phenylketonuria (PKU) is a genetic disorder caused by a


mutation in the gene for an enzyme, which results in the elimination of enzymatic
activity. The question asks the examinee to consider whether an antisense drug could
help individuals who have PKU. An antisense drug works to prevent the expression of
undesirable genes but does nothing to remedy the problem of a gene that produces
an ineffective product. The only way to cure PKU would be to add a gene or gene
product that could lead to the production of effective enzymes to replace the ones
that do not function correctly. D is the best answer because a blockage of gene
expression would only prevent the production of defective enzymes but would not
remedy the problem caused by the absence of effective enzymes. A and B are
incorrect because preventing expression of an inactive protein is not a cure. C is
incorrect because the stability of the mRNA is irrelevant to the situation. Thus, D is
the best answer.

Solution

Guess
209. An effective and efficient method for the delivery of an antisense gene could
be:

A
) orally as an emulsified product.

B
) microinjection into individual body cells.

C
) intravenously as a nonantigenic, blood-stable product.

D
) infection of an embryo by a virus modified to carry the gene.

The question asks the examinee to identify an effective and efficient method for the
delivery of an antisense gene. For an antisense gene to work, it must be incorporated
into the cell in which it will perform its job so that its product is available to hybridize
with the sense mRNA that needs to be blocked. Of the options listed, the best way to
deliver the antisense gene into all the cells of the individual would be to infect an
embryo with a virus that carries the antisense gene. The appropriate virus could
become incorporated into the genome of the embryonic cells, thus causing all cells
derived from these embryonic cells to contain the antisense gene. Therefore, D is the
best answer. A is incorrect because the gene would most likely be destroyed in the
acidic environment of the stomach. B is not the best option because microinjection of
a gene into all cells would be extremely impractical. C is incorrect because even
though the gene could be injected as a blood-stable product, it is not likely that the
drug would enter its intended target cells. Thus, D is the best answer.

Solution

Guess
211. Which of the following describes a primary function of the myelin sheath?

A
) It provides nutrients to motor neurons.

B
) It regulates synaptic vesicle discharge.

C
) It guides dendrite growth and branching.

D
) It increases the rate of conduction of action potentials.

The question asks the examinee to identify a primary function of the myelin sheath.
The myelin sheath is a fatty layer surrounding the long axons of neurons in
vertebrates. A primary function of this layer is to enable nerve impulses (action
potentials) to travel more quickly by jumping from node to node rather than traveling
continuously along the entire length of the neuron. This supports D, which states that
the myelin sheath increases the rate of conduction of action potentials. A, B, and C
are incorrect because they describe roles that the myelin sheath does not provide.
Thus, D is the best answer.
Solution

Guess
212. When muscles in the skin contract and cause the hair of an animal to “stand
on end,” the skin could be functioning as a regulator of:

A
) pH.

B
) salt excretion.

C
) body temperature.

The question asks the examinee to identify a likely regulatory function of an animal’s
skin when its muscles contract, causing the animal’s hair to stand on end. A major
function of hair is to help mammals regulate body temperature by reducing the
amount of heat lost to the environment. Hair provides insulation. When the hair
stands on end, trapping air within the layer of hair, then its insulating effect becomes
even more effective. Thus, as C indicates, a primary function of this action is to help
regulate body temperature. The contraction of skin muscles that causes the hair to
stand on end would have little effect on skin pH (A), salt excretion (B), or skeletal
muscle tone (D). Thus, C is the best answer.

D
) skeletal muscle tone.

Solution

Guess
213.

Of the isomeric alcohols (compounds 1–4), which is most reactive in an SN1 reaction
with HBr?

A
) Compound 1

B
) Compound 2

C
) Compound 3

D
) Compound 4

The most reactive alcohol in an SN1 reaction is the one that will generate the most
stable intermediate carbocation. The 3º carbocation generated by Compound 4 is the
most stable, and therefore Compound 4 will be the most reactive. Thus, D is the best
answer.

Solution

Guess
214. If chromosomal duplication before tetrad forma-tion occurred twice during
spermatogenesis, while the other steps of meiosis proceeded normally, which of the
following would result from a single spermatocyte?

A
) One tetraploid sperm

B
) Four diploid sperm

The question asks the examinee to predict the result if chromosomal duplication
occurred twice before tetrad formation, but if all other steps in spermatogenesis
occurred normally. If all other steps occur normally, then four sperm would be
produced. Because replication occurred twice instead of once prior to tetrad
formation, each sperm would have twice the normal amount of DNA. As a result, four
diploid sperm would be produced instead of four haploid sperm. Therefore, B is the
best answer. A, C, and D are incorrect because they describe results that contain an
incorrect number of sperm or ploidy number for the sperm produced. Thus, B is the
best answer.

C
) Four haploid sperm

D
) Eight haploid sperm

Solution

Guess
215. All of the following occur during normal inspiration of air in mammals EXCEPT:

A
) elevation of the rib cage.
B
) relaxation of the diaphragm.

The question asks the examinee to identify which one of four actions is not
associated with normal inspiration in mammals. B is the best answer because the
diaphragm does not relax during inspiration. The diaphragm contracts and pulls
downward, causing air to enter the lungs. A, C, and D all describe actions that do
occur during inspiration. Elevation of the rib cage increases the volume inside the
chest cavity (A), reduction of pressure inside the pleural cavity causes air to move
into the lungs (C), and contraction of the external intercostal rib muscles helps the
chest expand (D). Thus, B is the best answer.

C
) reduction of pressure in the pleural cavity.

D
) contraction of the external intercostal rib muscles.

Solution

Guess
216. Radioactively labeled uracil is added to a culture of actively dividing
mammalian cells. In which of the following cell structures will the uracil be
incorporated?

A
) Chromosomes

B
) Ribosomes

The question asks the examinee to predict the cell structure in which radioactively
labeled uracil will become incorporated if added to a culture of actively dividing
mammalian cells. Uracil is a component of RNA. Therefore, one would expect to find
the radioactively labeled uracil in cell structures that contain RNA. B is the best
answer because ribosomes contain rRNA and proteins. A is incorrect because
chromosomes, by definition, consist primarily of proteins and DNA. Although DNA is
composed of nucleic acids, DNA contains thymine instead of uracil. C and D are
incorrect because RNA is not an integral component of either lysosomes (C) or the
nuclear membrane (D). Thus, B is the best answer.

C
) Lysosomes

D
) Nuclear membrane

You might also like